• Menlo Coaching
  • Practice Questions
  • GMAT Critical Reasoning Sample Questions
  • Sign up for GMAT Prep

Memorial Day Sale on GMAT Prep! 20% Off GMAT Prep Course Enrollment (Ends May 28)

Official Sample GMAT Critical Reasoning Questions

We’ve already covered why studying with official practice questions is the best way to prepare for the GMAT . But even if you come up with the correct answer to an official problem, you still might not understand the underlying principles used to create that particular question, leaving yourself open to traps and pitfalls set by the test writers. In the explanations below, I will use some of the core tenets of the Menlo Coaching GMAT curriculum to breakdown two official GMAT critical reasoning practice questions and provide important principles for correctly attacking this question type in the future. 

For most students, critical reasoning is a completely new exercise—a question type never before seen or practiced. The instructions in the GMAT Official Guide state: “Answering Critical Reasoning questions requires no specialized knowledge. You don’t need to know the terminology or conventions of formal logic.” While technically this may be true, it certainly doesn’t hurt to have a formal understanding of common logical fallacies and to be comfortable with terminology!

GMAT Critical Reasoning, Sample Question #1

Which of the following most logically completes the argument? A new machine for harvesting corn will allow rows to be planted only fifteen inches apart, instead of the usual thirty inches. Corn planted this closely will produce lower yields per plant. Nevertheless, the new machine will allow corn growers to double their profits per acre because ________.

  • with the closer spacing of the rows, the growing corn plants will quickly form a dense canopy of leaves, which will, by shading the ground, minimize the need for costly weed control and irrigation
  • with the closer spacing of the rows, corn plants will be forced to grow taller because of increased competition for sunlight from neighboring corn plants
  • with the larger number of plants growing per acre, more fertilizer will be required
  • with the spacing between rows cut by half, the number of plants grown per acre will almost double
  • with the closer spacing of the rows, the acreage on which corn is planted will be utilized much more intensively than it was before, requiring more frequent fallow years in which corn fields are left unplanted

GMAT Critical Reasoning, Sample Question #2

Editorial in Krenlandian Newspaper: Krenland’s steelmakers are losing domestic sales because of lower-priced imports, in many cases because foreign governments subsidize their steel industries in ways that are banned by international treaties. But whatever the cause, the cost is ultimately going to be jobs in Krenland’s steel industry. Therefore, it would protect not only steel companies but also industrial employment in Krenland if our government took measures to reduce cheap steel imports.

Which of the following, if true, most seriously weakens the editorial’s argument?

  • Because steel from Krenland is rarely competitive in international markets, only a very small portion of Krenlandian steelmakers’ revenue comes from exports.
  • The international treaties that some governments are violating by giving subsidies to steelmakers do not specify any penalties for such violations.
  • For many Krenlandian manufacturers who face severe international competition in both domestic and export markets, steel constitutes a significant part of their raw material costs.
  • Because of advances in order-taking, shipping, and inventory systems, the cost of shipping steel from foreign producers to Krenland has fallen considerably in recent years.
  • Wages paid to workers in the steel industry in Krenland differ significantly from wages paid to workers in many of the countries that export steel to Krenland.

Final Thoughts: Critical Reasoning Questions in GMAT Prep

To succeed on GMAT critical reasoning questions, it is essential that you learn defined strategies and best practices, yet most people who study on their own do not do that. They simply use their smarts and logic to answer these questions, an approach that breaks down on harder examples in the time-pressured exam environment. As you can see with the other sample questions , you must learn how GMAT critical reasoning questions are made difficult and what mindsets and strategies allow you to overcome this difficulty. If you look carefully at this well-crafted question below from the official practice tests, you should see a very similar scenario to the “shiny penny” con as described in our GMAT data sufficiency sample questions and explanations . The same mindset and thought process is tested over and over in this exam through all the different question types! Keep this in mind as you pursue your GMAT prep, no matter what form that prep takes.

Revisit GMAT Critical Reasoning preparation with our GMAT prep course online , offering expertly analyzed sample questions.

Sample GMAT Questions by Topic

  • GMAT Problem Solving Questions
  • GMAT Data Sufficiency Questions
  • GMAT Data Insights Questions
  • GMAT Reading Comprehension Questions

Book a GMAT Strategy Call with Hailey

gmat critical thinking sample questions

Replies in 24 hours

[email protected]

✓ Get feedback on your profile as a test-taker

✓ Ask questions about strategy, timelines, or the GMAT itself

✓ Learn what you need to work on to improve your score

✓ Find out whether you’d be a fit with live classes or private tutoring

  • Skip to content

GMAT Prep Courses & Tutoring

Free GMAT Practice Questions

Practice makes perfect—or at least that's how the old saying goes—and it certainly applies to our free GMAT practice questions. Our Free Practice Questions are designed to give you the thorough understanding of how to go about solving a problem that you crave. Our thorough explanations show you what to expect from each GMAT question, detailing question-specific hurdles and common traps. Thankfully, our practice questions provide a wide variety of question types spanning across all sections, securing an abundance of insight-turned-strategy you can implement on test day to turn into high-scoring gold.

Manhattan Review prides itself in providing quality free practice questions to all prospective students, so please take a crack at the 52 free questions we have to offer as a courtesy to all GMAT learners. In the end, the only way to find out where you need your score to be is to discover where it currently is. Take advantage of this free resource that's sure to help you along your way to a high score.

You have not answered any question so far. You can answer all questions in a row (click on "All Questions") or only all questions of a particular section (click on that Section) or a single selected question (click on that Question).

GMAT is a registered trademark of the Graduate Management Admission Council (GMAC), which is unaffiliated with and does not endorse this website.

gmat critical thinking sample questions

Homepage > GMAT Preparation > GMAT Critical Reasoning – What it tests + Sample Practice CR Questions

GMAT Critical Reasoning – What it tests + Sample Practice CR Questions

Posted by Harsha R | Aug 9, 2022 | GMAT CR , GMAT Preparation , GMAT Verbal

GMAT Critical Reasoning – What it tests + Sample Practice CR Questions

This article provides a complete overview of the GMAT CR. You will learn everything you need to know about GMAT Critical Reasoning (or GMAT Logical reasoning, as it is commonly known), what it tests, the skills required to ace it, and a whole list of different GMAT CR sample questions for your practice!

GMAT Critical Reasoning Sample Questions

“Magic (on the GMAT) cannot happen without logic (reasoning).” – Your friendly, neighbourhood e-GMAT Verbal Expert

GMAT Critical Reasoning (or CR, as it is ubiquitously known) is arguably the subsection of GMAT Verbal that best exemplifies the importance of logical reasoning on the test. Many of you are aware of it. Some of you love it. Some of you fear it. But if the GMAT is on your horizon, you should know how to ace it!

CR accounts for one-third of your verbal score. What this means is that if you have a lofty target GMAT score (700 or higher), you cannot take it lightly.

But what exactly is GMAT Critical Reasoning?

Well, GMAT CR is a test of our logical reasoning skills. As per the GMAT Official Guide, the GMAT tests our reasoning skills in three ways:

  • Our understanding of how an argument is constructed
  • Our skill in evaluating arguments
  • Our ability to formulate or evaluate a plan of action

We have only scratched the surface here. Do you want to understand GMAT Critical Reasoning at a deeper level?

If yes, you should read on. In this article, we will discuss the basics of Critical Reasoning – what it really is, what it tests, and the various question types.

Let’s jump straight in.

2. Why should you listen to us?

3. structure of a critical reasoning question, construction, logically completes (fill in the blanks), other structure/reasoning-based questions, plan questions, 6. your next steps | gmat critical reasoning.

You may be wondering why you should listen to what we have to say on all things GMAT CR.

Well, for starters, our strategies for CR have helped us deliver stellar success in the GMAT universe over the years.

We have been able to help students with low starting CR abilities excel in CR. For example, using Prethinking (our flagship CR strategy!), Jim , a current student at Booth, improved his CR score from the 39 th percentile to the 94 th percentile.

GMAT CR Sample Questions

We have also been able to help students with high starting CR abilities get even better. Here are a couple of examples – Shaarang improved from a 640 to a 740 in twelve days (V35 to V42), and Magdalena improved from a 680 to a 780 (V38 to V48).

In fact, since 2020, e-GMATers (our students!) have reported more 700+ scores than have students from any other test prep company. This article will give you a peek into how we have been able to do this!’

GMAT Critical Reasoning Overview

Now, let’s explore the world of CR.

Here is what a CR Question looks like (this is an example from the Official Guide):

gmat critical thinking sample questions

Every GMAT Critical Reasoning question consists of three components – the passage/argument, the question stem, and the answer choices .

The passage/argument, also called the Stimulus ,is the piece of text given to us, which we are meant to understand and analyse. A passage simply provides information, whereas an argument has both some stated information (premises) and a defined main conclusion (the central claim made by the author of the text, which is based on some (or all) of the stated information).

Every CR question consists of three components – the passage/argument, the question stem, and the answer choices.

The question stem tells us what it is we need to do. And then, there are five answer choices.

A CR passage/argument is typically less than 100 words in length. Answer choices typically range from below 15 words to up to 35 words in length (each!). Overall, this means that a CR question typically has between 120-200 words of text.

An important point to note: As with other verbal subsections, there is only one correct answer among the five answer choices. So, if you ever get the feeling that more than one answer is surely correct, that is a red flag.

If you ever get the feeling that more than one answer is surely correct, that is a red flag.

Before you proceed further, here is a quick quiz to determine your current CR abilities and get a feel of the official CR questions tested on the GMAT. Try your hand at the same:

At this point, if you are itching to try your hand at a few more CR questions, we have got you covered. Our free trial contains a good collection of CR questions for your practice.

4. What exactly is tested on GMAT CR?

First, let us talk about what GMAT CR does not test.

The questions we see on GMAT Critical Reasoning cover various topics such as politics, art, science, literature, psychology, etc. But we are not being tested on the subject matter of these topics. The GMAT, at best, expects us to apply what is common knowledge or generally known. One need not have domain-specific knowledge to solve CR questions. Simply put, one need not be a specialist in psychology to be able to solve a CR question pertaining to the topic.

GMAT CR cover various topics such as politics, art, science, literature, psychology, etc. One need not have domain specific knowledge to solve CR questions.

Now, to understand what GMAT CR tests, first let us understand the term “reasoning”.

As per the Official Guide, any set of statements in which at least one of the statements provides logical support to another will be considered an example of “reasoning ”.

Here is a quick example:

The sun is not a planet. Therefore, it must be a star.

The first statement is used as “reasoning” to support the second statement.

The above example is one of an argument, with the first statement being a premise and the second being the conclusion drawn based on the premise.

Now, a CR passage need not always contain reasoning. It may simply present information. For example:

The sun is not a planet. Only a star or a planet can have the kind of size the sun has.

Here, both statements merely provide us with certain information. Neither statement is used as reasoning for the other. But in such a case, the question would typically ask us to arrive at a conclusion that uses the given information as reasoning.

For example: If only a star or a planet can have the kind of size the sun has, then the sun must be a planet or a star. But we are given that the sun is not a planet. Therefore, we can logically conclude or reason out that the sun must be a star .

In other words, irrespective of whether the passage text itself contains reasoning, CR questions involve an assessment of our reasoning skills.

But what exactly is assessed under the umbrella of reasoning skills?

The skills pertaining to reasoning that are tested by the GMAT Critical Reasoning are divided into four categories – Analysis, Construction, Critique, and Plan.

Here are the different CR question types that test these skills:

Different CR question types that test CR skills

Here, we need to analyse an argument. This involves understanding the logical reasoning/structure of the argument and the relationships between various parts of the argument.

Here, we are tested on our ability to construct solid arguments. For example – finding what can be logically concluded/inferred or finding what new information would be needed for the argument to hold true.

Here, we are tested on our ability to question the validity of arguments, identify how to strengthen or weaken arguments, identify the flaw in an argument, etc. In other words, we are critiquing a given argument.

Plan questions test our ability to construct and critique arguments pertaining to a plan of action. For example – a question may ask us to find the flaw in a plan, or maybe the underlying assumption for a plan to work. Plan questions, from a conceptual standpoint, are either construction questions or critique questions.

The skills pertaining to reasoning that are tested by the GMAT are divided into four categories – Analysis, Construction, Critique and Plan.

Now that we know what are the specific skills that get tested on the GMAT, let us now look at the various question types that are designed to assess us on these skills.

5. GMAT Critical reasoning question types

The major GMAT Critical Reasoning question types can be categorized, based on the skills being tested, as follows:

Let us understand what these question types are, one by one.

In Inference questions, we are asked to find a statement that can be logically concluded or reasoned out as certainly true, based on the information given in the passage.

Inference Question – GMAT CR Sample Question #1

A company’s two divisions performed with remarkable consistency over the past three years: in each of those years, the pharmaceuticals division has accounted for roughly 20 percent of dollars sales and 40 percent of profits, and the chemicals division for the balance. Regarding the past three years, which of the following can properly be inferred from the statement above? (A) Total dollars sales for each of the company’s divisions have remained roughly constant. (B) The pharmaceuticals division has faced stiffer competition in its markets than has the chemicals division. (C) The chemical division has realized lower profits per dollars of sales than has the pharmaceuticals division. (D) The product mix offered by each of the company’s divisions has remained unchanged. (E) Highly profitable products accounted for a higher percentage of the chemicals division’s sales than of the pharmaceuticals division’s

In Assumption questions, we are asked to identify an assumption the author of the argument must have made in order to claim something (conclusion), based on some stated information (premise(s)).

If the assumption is not true, the author cannot make that conclusion anymore (i.e., the conclusion is not valid anymore).

An important point about assumptions: The author’s conclusion depends on the assumption statement being true. If the assumption is not true, the author cannot make that conclusion anymore (i.e., the conclusion is not valid anymore).

Assumption Question – GMAT Critical Reasoning Sample Question #2

Ythex has developed a small diesel engine that produces 30 percent less particulate pollution than the engine made by its main rival, Onez, now widely used in Marania; Ythex’s engine is well-suited for use in the thriving warehousing businesses in Marania, although it costs more than the Onez engine. The Maranian government plans to ban within the next two years the use of diesel engines with more than 80 percent of current diesel engine particulate emissions in Marania, and Onez will probably not be able to retool its engine to reduce emissions to reach this target. So if the ban is passed, the Ythex engine ought to sell well in Marania after that time.

Which of the following is an assumption on which the argument above depends?

A. Marania’s warehousing and transshipment business buys more diesel engines of any size than other types of engines. B. Ythex is likely to be able to reduce the cost of its small diesel engine within the next two years. C. The Maranian government is generally favorable to anti-pollution regulations. D. The government’s ban on high levels of pollution caused by diesel engines, if passed, will not be difficult to enforce. E. The other manufacturers of small diesel engines in Marania, if there are any, have not produced an engine as popular and clean-running as Ythex’s new engine.

An evaluation is an assessment. Evaluating an argument means assessing the validity and strength of an argument, i.e., assessing whether the conclusion follows logically from the given premise(s).

An Evaluate question asks us to identify, from among the answer choices, what will best help in evaluating/assessing whether a certain conclusion is logically solid.

Every Evaluate answer choice, irrespective of how it is written, poses a question. These questions have two possible answers – Yes or No. Here is an official Evaluate question for your reference.

Evaluate Question – GMAT CR Sample Question #3

Scientists have modified feed corn genetically, increasing its resistance to insect pests. Farmers who tried out the genetically modified corn last season applied less insecticide to their corn fields and still got yields comparable to those they would have gotten with ordinary corn. Ordinary corn seed, however, costs less, and what these farmers saved on insecticide rarely exceeded their extra costs for seed. Therefore, for most feed-corn farmers, switching to genetically modified seed would be unlikely to increase profits.

Which of the following would it be most useful to know in order to evaluate the argument? (A) Whether there are insect pests that sometimes reduce feed-corn yields, but against which commonly used insecticides and the genetic modification are equally ineffective (B) Whether the price that farmers receive for feed corn has remained steady over the past few years (C) Whether the insecticides typically used on feed corn tend to be more expensive than insecticides typically used on other crops (D) Whether most of the farmers who tried the genetically modified corn last season applied more insecticide than was actually necessary (E) Whether, for most farmers who plant feed corn, it is their most profitable crop

gmat critical thinking sample questions

Observe how every answer choice poses a question with a simple Yes/No answer.

Important: A correct answer choice will increase our belief in the conclusion in one direction ( For example: YES, to the question posed by the answer choice) and reduce our belief in the conclusion in the other direction (For example: NO, to the question posed by the answer choice).

In Strengthen questions, we are asked to identify a statement that brings in new information which supports the conclusion of an argument. Here, we are looking for a statement that increases our belief in that conclusion.

Strengthen Question – GMAT Critical Reasoning Sample Question #4

A fossil recently discovered in Marlandia, a chain of islands, proves that a present-day reptile indigenous to Marlandia is descended from an ancient reptile species that lived on the islands millions of years ago. The finding is surprising since the ancestral species was thought to have become extinct when Marlandia was submerged in a global sea-level rise twenty-five million years ago. Based on the new discovery, many scientists have concluded that the sea-level rise in question left at least part of Marlandia unsubmerged.

Which of the following would, if true, provide the most additional support for the scientists’ conclusion? A. Reptiles in Marlandia have adapted to many environmental changes since the sea-level rise. B. Marlandia separated from a much larger landmass about eighty million years ago. C. No fossils that prove the relationship between the present-day species and the ancestral species have been found anywhere other than Marlandia. D. The present-day reptiles are able to thrive on very tiny Marlandia islands. E. The ancestral reptiles could not have survived long at sea.

In Weaken questions, we are asked to identify a statement that brings in new information which casts a doubt over the conclusion of an argument. Here, we are looking for a statement that reduces our belief in that conclusion.

Weaken Question – GMAT CR Sample Question #5

Advertisement: Our competitors’ computer salespeople are paid according to the value of the products they sell, so they have a financial incentive to convince you to buy the most expensive units—whether you need them or not. But here at Comput-o-Mart, our salespeople are paid a salary that is not dependent on the value of their sales, so they won’t try to tell you what to buy. That means when you buy a computer at Comput-o-Mart, you can be sure you’re not paying for computing capabilities you don’t need. Which of the following would, if true, most weaken the advertisement’s reasoning? A. Some less-expensive computers actually have greater computing power than more expensive ones. B. Salespeople who have a financial incentive to make sales generally provide more attentive service than do other salespeople. C. Extended warranties purchased for less-expensive computers can cost nearly as much as the purchase price of the computer. D. Comput-o-Mart is open only limited hours, which makes it more difficult for many shoppers to buy computers there than at other retail stores. E. Comput-o-Mart does not sell any computers that support only basic computing.

These questions ask us to find the statement among the answer choices that highlights a logical flaw/problem in an argument’s reasoning.

Flaw Question – GMAT Critical Reasoning Sample Question #6

The contingency-fee system, which allows lawyers and their clients to agree that the lawyer will be paid only in the event of success, does not increase the number of medical malpractice lawsuits brought against doctors. As attorneys must cover the costs for their time and research, they want to be assured that any medical malpractice case they accept on a contingency-fee basis has substantial merit. Consequently, attorneys turn away many people who come to see them, for lack of a good case. The argument above is most vulnerable to criticism on the grounds that it fails to A. specify the criteria attorneys use to judge the merits of a medical malpractice case B. consider whether, in the absence of a contingency-fee option, even people with meritorious cases are much less likely to initiate litigation if they believe they might incur large legal fees C. note whether, in successful medical malpractice lawsuits, the average monetary award after legal costs have been deducted is less under contingency-fee arrangements than otherwise D. consider the effect of the contingency-fee system on the number of lawsuits sought for reasons other than medical malpractice E. acknowledge the rising cost of medical malpractice insurance

In these questions, the passage/argument is incomplete: it contains a blank. The question will ask us to fill in the blank, i.e., logically complete the blank portion of the passage/argument.

Logically Completes Question – GMAT CR Sample Question #7

Which of the following best completes the passage below?

When the products of several competing suppliers are perceived by consumers to be essentially the same, classical economics predicts that price competition will reduce prices to the same minimal levels and all suppliers’ profits to the same minimal levels. Therefore, if classical economics is true, and given suppliers’ desire to make as much profit as possible, it should be expected that ____________ (A) in a crowded market widely differing prices will be charged for products that are essentially the same as each other (B) as a market becomes less crowded as suppliers leave, the profits of the remaining suppliers will tend to decrease (C) each supplier in a crowded market will try to convince consumers that its product differs significantly from its competitors’ products. (D) when consumers are unable to distinguish the products in a crowded market, consumers will judge that the higher-priced products are of higher quality (E) suppliers in crowded markets will have more incentive to reduce prices and thus increase sales than to introduce innovations that would distinguish their product from their competitors’ products

Now, there are typically two types of questions here.

Logically Completes – Inference

Here, the given statements are merely information. The blank we need to fill with the correct answer choice is what can be logically concluded or reasoned out based on the given statements. Hence, this is practically an Inference question.

Logically Completes – Strengthen

Here, the given statements include a stated main conclusion. The blank we need to fill with the correct answer choice is some additional premise that supports or strengthens the conclusion being made. Hence, this is practically a Strengthen question.

A paradoxical situation is one that seems to not make logical sense, and therefore seems contradictory to what was logically expected.

In this question, we are tasked with finding an explanation that can “resolve the paradox”. Simply put, our job is to find a logical explanation for the paradoxical situation.

Paradox Question – GMAT Critical Reasoning Sample Question #8

When new laws imposing strict penalties for misleading corporate disclosures were passed, they were hailed as initiating an era of corporate openness. As an additional benefit, given the increased amount and accuracy of information disclosed under the new laws, it was assumed that analysts’ predictions of corporate performance would become more accurate. Since the passage of the laws, however, the number of inaccurate analysts’ predictions has not in fact decreased. Which of the following would, if true, best explain the discrepancy outlined above?

A) The new laws’ definition of “misleading information” can be interpreted in more than one way. B) The new laws require corporations in all industries to release information at specific times of the year. C) Even before the new laws were passed, the information most corporations released was true. D) Analysts base their predictions on information they gather from many sources, not just corporate disclosures. E) The more pieces of information corporations release, the more difficult it becomes for anyone to organize them in a manageable way.

In this question type, we are given an argument (premises + main conclusion). Two of the statements in the argument are highlighted in bold . We are asked to identify the functions played by the two boldfaced statements in the argument.

Here is a quick official example of what a Boldface question looks like.

gmat critical thinking sample questions

Boldface – GMAT CR Sample Question #9

Most of Western music since the Renaissance has been based on a seven-note scale known as the diatonic scale, but when did the scale originate? A fragment of a bone flute excavated at a Neanderthal campsite has four holes, which are spaced in exactly the right way for playing the third through sixth notes of a diatonic scale.  The entire flute must surely have had more holes,  and the flute was made from a bone that was long enough for these additional holes to have allowed a complete diatonic scale to be played. Therefore,  the Neanderthals who made the flute probably used a diatonic musical scale.

In the argument given, the two portions in boldface play which of the following roles?

A. The first is presented as evidence that is confirmed by data presented elsewhere in the argument given; the second states a hypothesis that this evidence is used to undermine. B. The first is an opinion, for which no supporting evidence is presented in the argument given, that is used to support the main conclusion of the argument; the second is that main conclusion. C. The first describes a discovery as undermining the position against which the argument is directed; the second states the main conclusion of the argument. D. The first is a preliminary conclusion drawn on the basis of evidence presented elsewhere in the argument given; the second is the main conclusion that this preliminary conclusion supports. E. The first provides evidence to support the main conclusion of the argument; the second states a subsidiary conclusion that is drawn in order to support the main conclusion stated earlier in the argument.

There are some other miscellaneous question types that are all about understanding the line of reasoning or understanding the logical structure given in the passage/argument. These are comparatively rarer on the GMAT, but here are a few for your reference:

Reasoning – GMAT Critical Reasoning Sample Question #10

Networks of blood vessels in bats’ wings serve only to disperse heat generated in flight. This heat is generated only because bats flap their wings. Thus paleontologists’ recent discovery that the winged dinosaur Sandactylus had similar networks of blood vessels in the skin of its wings provides evidence for the hypothesis that Sandactylus flew by flapping its wings, not just by gliding. In the passage, the author develops the argument by (A) forming the hypothesis that best explains several apparently conflicting pieces of evidence (B) reinterpreting evidence that had been used to support an earlier theory (C) using an analogy with a known phenomenon to draw a conclusion about an unknown phenomenon (D) speculating about how structures observed in present-day creatures might have developed from similar structures in creatures now extinct (E) pointing out differences in the physiological demands that flight makes on large, as opposed to small, creatures

Analogous Reasoning/Parallel Reasoning

In this question, the given passage has a certain type of reasoning structure. We have to select, from the answer choices, a similar reasoning structure in a completely different scenario (not related to the passage!). Sounds interesting? Here is an example of this question type.

Identifying the logical structure

In this question, the given passage usually presents a conversation between two or more individuals. We need to analyse all arguments, understand their logical structure, and identify how a speaker supports or counters another’s view.

Here is an example for this question type.

As highlighted earlier, plan-based questions, at a concept level, are either construction questions or critique questions.

Plan-based questions, at a concept level, are either construction questions or critique questions.

Here, there is a given plan of action discussed, which has a specific goal. Here are some possible question sub-types:

  • What can be logically concluded about the plan to do X? (Inference)
  • For the plan Y to succeed, what must be true? (Assumption)
  • Which of the following statements most strongly supports the notion that the plan Y will succeed in doing Z? (Strengthen)
  • Which of the following statements helps us most in evaluating if the plan Y will be successful? (Evaluate)
  • The plan to do X is most vulnerable to which of the following criticisms? (Flaw)

Here is an example of a plan-type CR question for your reference.

At this point, you may be wondering – which of these question types are most commonly seen on the GMAT?

Assumption, Strengthen, and Weaken are arguably the most common question types on the test. Of course, this does not necessarily mean that you will get mostly these question types. Every individual’s experience on the test is unique (After all, the GMAT is an adaptive test!).

GMAT Critical Reasoning Practice Questions- List

Here is a list of all the CR practice questions discussed in this article, for your quick reference:

  • Practice Question 1: Inference
  • Practice Question 2: Assumption
  • Practice Question 3: Evaluate
  • Practice Question 4: Strengthen
  • Practice Question 5: Weaken
  • Practice Question 6: Flaw
  • Practice Question 7: Logically completes
  • Practice Question 8: Paradox
  • Practice Question 9: Boldface
  • Practice Question 10: Reasoning

There you go!

I hope this article was useful in getting you acquainted with GMAT Critical Reasoning. In subsequent articles, we will explore the world of GMAT CR further – what skills does one need to build to ace CR, how to prepare, etc. Make sure you catch those articles too!

If you would like to try attempting some CR questions to get a feel for CR, you can check out our free trial.

Through these files, you will get introduced to Prethinking, the CR strategy that has played a part in the success of thousands of students. If you want to explore Prethinking further, you may want to check out our free webinar on Prethinking for CR Assumption questions.

Hope this helps! Harsha

About The Author

gmat critical thinking sample questions

Don't limit yourself!

Don't miss these.

gmat critical thinking sample questions

Helpful posts Curated just for you!

gmat critical thinking sample questions

You might also like

gmat critical thinking sample questions

Browse Related Topics

gmat critical thinking sample questions

Achieve 685+ on the GMAT in 30 days!

Sign up for our free trial and get.

gmat critical thinking sample questions

400+ Practice questions with detailed solutions

10+ hours of ai-driven video lessons, adaptive mock test with osr+ analysis.

Begin your GMAT Prep today!

GMAT Prep Online Guides and Tips

8 expert tips for gmat critical reasoning questions.

gmat critical thinking sample questions

Critical reasoning questions often worry GMAT test-takers. What exactly are GMAT critical reasoning questions, and what skills do you need to develop to answer them correctly? How can you best prepare for critical reasoning questions before the exam?

In this article, I’ll go over the five main critical reasoning question types and examples of each one, as well as plenty of GMAT critical reasoning tips on how to ace these question types come exam day.

GMAT Critical Reasoning Overview

Critical reasoning questions appear in the verbal section of the GMAT , alongside sentence correction questions and reading comprehension questions. There are 36 questions in total on the verbal section, and around 11 of them are critical reasoning questions .

GMAT critical reasoning questions test your ability to evaluate and draw conclusions based on logical arguments. You won’t need any formal training in logic or rhetoric to do well on critical reasoning questions. However, you will need to be able to analyze an argument and unpack it into its various parts.

The 5 GMAT Critical Reasoning Question Types

There are five main critical reasoning question types: Strengthen the Argument, Weaken the Argument/Find the Flaw, Inference, Draw a Conclusion, Find the Assumption, and Paradox/Discrepancy. Let’s go through each question type. For each question type, we’ll go over what the question will be asking you to do, key words associated with that type, how each it will be worded, and an example question.

Not sure how or what to study? Confused by how to improve your score in the shortest time possible? We've created the only Online GMAT Prep Program that identifies your strengths and weaknesses, customizes a study plan, coaches you through lessons and quizzes, and adapts your study plan as you improve.

We believe PrepScholar GMAT is the best GMAT prep program available , especially if you find it hard to organize your study schedule and don't want to spend a ton of money on the other companies' one-size-fits-all study plans.

     Improve Your GMAT Score by 60 Points, Guaranteed     

Question Type 1: Strengthen the Argument

Strengthen the Argument questions will ask you to choose the statement or piece of evidence that would support, bolster, or add to the argument in the given passage. Start by figuring out the main argument of the passage. Then choose the answer choice that provides the best evidence for that argument.

Key words you might see are support or strengthen. Clue words can help you figure out what category a question falls into, which will help you know how to approach the question.

A Strengthen the Argument question may be worded as:

Which of the following, if true, most strongly supports…?

Which of the following, if true, would most strengthen…?

The statements above, if true, best support which of the following assertions?

supports

The answer you choose for a Strengthen the Argument question should be directly relevant to the specific argument in question and should provide a specific piece of information that justifies the argument (in this case, the Write Company’s assertion that their plan will lead to an increase in its sales of pencil leads).

Answer choice A doesn’t provide enough directly relevant info; we are told that the Write Company’s pencil leads are more expensive than other manufacturers’, but don’t know whether their pencils themselves are more or less expensive than average.

Answer choice B is too general, as it addresses annual sales of all mechanical pencils and not Write Company sales specifically.

The passage tells us that the new lead will be sold at the same price as the Write Company’s current lead, so answer choice C doesn’t tell us anything about the potentiality of increased sales.

Answer choice D tells us only about a rival manufacturer, which may or may not have an impact on the Write Company’s sales.

Answer choice E is correct, as this test marketing information strongly supports the Write Company’s projection of the success of its new plan.

Unfortunately, this won't be one of your answer choices.

Weaken the Argument/Find the Flaw

Weaken the Argument questions will ask you either to 1) select the fact or piece of evidence that would most effectively undermine the given argument or 2) to find the logical flaw in the given argument’s reasoning. If you know you’re being asked a Weaken the Argument question, start by figuring out (and perhaps jotting down) what exactly that argument is. Then look at each answer choice carefully. If true, does it create a crack in the argument’s foundation?

Key words you might see are weaken or doubts.

A Weaken the Argument question may be worded as:

Which of the following, if true, most seriously weakens the argument?

Which of the following, if true, would cast the most doubt on…?

Want to improve your GMAT score by 60 points?

We have the industry's leading GMAT prep program. Built by Harvard, MIT, Stanford, and Wharton alumni and GMAT 99th percentile scorers, the program learns your strengths and weaknesses and customizes a curriculum so you get the most effective prep possible.

Try PrepScholar GMAT for 5 Days Risk-Free.

The objection implied above…is based on doubts about…?

Which of the following points to the most serious logical flaw in the author’s argument?

weaken

Answer choice A doesn’t weaken the argument because the argument concerns whether or not the statue is a forgery, not how the museum obtained it.

Answer choice B fails to take into account the fact that any forger would likely imitate the most common features of Greek statues of the time period.

Answer choice C is correct. It gives a possible legitimate explanation for the statue’s uniform surface.

Answer choice D isn’t relevant because the statue under discussion has a uniform surface, not a weathered one.

Answer choice E provides an example that strengthens the argument rather than weakening it.

Can you spot the crack in the argument?

Inference questions ask you to make inferences—draw logical conclusions—based on the evidence in the given passage.

Key words you might see are imply or infer.

An Inference question may be worded as:

Which of the following can properly be inferred from the statements above?

Which of the following can be correctly inferred from the statements above?

Want to Identify YOUR GMAT Strengths and Weaknesses?

Our proprietary GMAT Diagnostic Assessment creates a customized study plan for you that takes you from registration all the way to test day! It is included with every account and proven to significantly maximize your score .

Get your personalized assessment as part of your 5 day risk-free trial now:

Get Your Free GMAT Diagnostic Assessment Here

Answer choice A only references underweight individuals, while the passage only specifically mentions overweight individuals, so we can’t make this inference based on the given information.

Answer choice B is incorrect. The passage only mentions the possibility of increasing HDL levels for some individuals through exercise and weight reduction. It doesn’t mention the individuals’ age or who is high-risk for high cholesterol levels.

Answer choice C makes a too-far leap in logic, generalizing about all human beings, while the passage only mentions the benefits of exercise and weight reduction for some. It also can’t be known from the passage whether exercise and weight reduction are the only or best methods of lowering cholesterol levels.

Answer choice D is correct. Since increased HDL levels decrease bloodstream cholesterol levels, and exercise and weight reduction increase HDL levels for some individuals, we can infer that regular exercise and weight reduction lowers cholesterol levels in the bloodstream of some individuals.

Answer choice E generalizes that ‘only’ exercise is necessary to decrease cholesterol levels. The passage doesn’t say that exercise is the only means of decreasing cholesterol levels, and it doesn’t mention the relative success of exercise programs for individuals of certain weights.

Inference questions ask you to look at the information presented in the given argument closely.

Find the Assumption

Find the Assumption questions will ask you to select the answer choice with the information that must be true (the ‘assumption’) in order for the given argument to be accurate. To figure out the answer, you’ll first need to figure out the main thrust of the given argument. The correct answer choice, again, will have to be true in order for that argument to be logical. Incorrect answer choices will often be possibly true, but won’t be absolutely necessary to the argument’s validity.

Key words you might see are assumption, because, or rely.

A Find the Assumption question may be worded as:

Which of the following is an assumption made in drawing the conclusion above?

Which of the following in an assumption on which the argument relies?

assumption

Answer choice A is too absolute. We can’t know for sure that a hiring program will always be successful if it includes interviews.

Answer choice B is off topic. The developments of job descriptions are not relevant to the argument.

Answer choice C is correct. For the interview to be an essential part of a successful hiring program, the interviewer must be able to accurately identify applicants whose personalities are unsuited to the job in question.

Answer choice D goes too far in its assumption. Note that extreme word ‘only.’

Answer choice E is incorrect—we can’t know this from the argument. We don’t know anything about past priorities in hiring decisions.

Learning to unpack arguments and identify premises, assumptions, and conclusions is an important skill for answering critical reasoning questions.

Paradox/Discrepancy

Paradox/Discrepancy questions will ask you to choose the answer choice that explains the paradox in the given argument. A paradox, in the context of the GMAT, refers to the coexistence of two seemingly contradictory pieces of information. The correct answer choice will logically explain why those pieces of information are not actually contradictory.

Key words you might see are explain, paradox, or discrepancy.

A Paradox question may be worded as:

Which of the statements below provides the most likely explanation for the two seemingly contradictory statements above?

Which of the following, if true, most helps to resolve the paradox outlined above?

Which of the following, if true, best explains the reason for the apparent discrepancy described above?

paradox

Answer choice A is correct and resolves the paradox. If brand names are believed by customers to be a guarantee of high quality, that explains their marketing advantage over rival brands.

Answer choice B wouldn’t resolve the paradox: If consumers understand that a brand name isn’t a guarantee of quality over time, the marketing advantage of a brand name should decrease rather than continue to increase.

Answer choice C is incorrect because corporations’ interest in brand names does not explain their marketing advantage.

Answer choice D doesn’t explain the paradox because the difficulty or ease of establishing a brand name isn’t relevant to its relative marketing advantage.

Answer choice E is also irrelevant: Transferring to a new advertising agency is not sufficient as an explanation for the marketing advantage experienced by all brand names.

Paradox questions ask you to resolve apparent contradictions, discrepancies or disparities.

Top GMAT Critical Reasoning Tips

Let’s go through five of the top GMAT critical reasoning tips to help you do your best on this tricky question type.

Pace yourself properly. You will have 65 minutes to complete the verbal section. GMAT critical reasoning questions, in general, don’t take quite as much time as the average reading comprehension question, but more time than a sentence correction question. As you work through your critical reasoning GMAT practice, you should plan to spend about a minute and a half on each one.

Read the question first. It can be helpful to read the question before you read the rest of the passage. Try to figure out which question type it is. This will help you figure out exactly what you should be looking for in the answer choices. For paradox questions, for example, you’re looking for a new piece of information that resolves a ‘problem’ or contradiction, whereas in an inference question you’re looking for a conclusion that can be reasonably drawn from the given information.

Think carefully about what you’re being asked. As soon as you read the question, try to discern what’s exactly is being asked of you. Are you being asked to draw a conclusion based on the given information? To reconcile two contradictory statements? This will determine how you approach the answer choices. For example, if you’re being asked to choose a statement that would weaken the argument presented in the passage, you’ll need to start off by 1) identifying the main argument of the passage and 2) select the answer choice that contradicts or provides an exception to that argument. If you’re being asked to select an answer choice that supports the main argument, you’ll still need to complete the first step (identifying the passage’s argument), but step 2 will be to select an answer choice that bolsters that argument.

Avoid extreme answer choices. Extreme or absolute answer choices are a red flag. Try to start taking notice of “clue words” in your critical reasoning GMAT practice. For the most part, avoid answer choices that contain language like ‘only,’ ‘never,’ ‘always,’ ‘all,’ ‘none,’ ‘best,’ ‘worst,’ ‘must,’ etc. They’re nearly always incorrect. For example, in answer choice C in the inference example above, “Exercise and weight reduction are the most effective methods” is a tip-off that the answer choice is likely wrong. Think about it: The passages you’re given in these questions are brief, and the accompanying questions ask you to do some nuanced critical thinking, so sweeping, broad generalizations are pretty unlikely candidates for correct answer choices.

Steer clear of answer choices with information that wasn’t directly given in the passage. This is especially true for inference questions. Everything you need to answer the question will be right there for you in the passage. If an answer choice is off topic or addresses a tangential issue (for instance, if an argument addresses the link between weight and age and an answer choice references the effects of exercise on weight), it might be a red flag.

Take note as you work through GMAT critical reasoning practice questions: Sometimes you’ll get new pertinent information in an answer choice (for example, the correct answer to a paradox question will provide an explanation for an apparent contradiction), but incorrect answer choices will contain information that’s not directly related to the passage at all or provide unrelated new information that doesn’t answer the question.

Pace yourself well on critical reasoning questions! Not answering every question will affect your score.

How to Prepare for GMAT Critical Reasoning Questions

Practice identifying premises, assumptions, and conclusions. Every argument has a premise (the foundational underlying logic of an argument, including the evidence behind a conclusion), one or more assumptions (information that must be true in order for the ultimate conclusion to be true) on which it’s based, and one or more possible conclusions (the statement that the premise supports). As you complete GMAT critical reasoning practice questions, label the premises, assumptions, and conclusions of each argument. This will help you learn to isolate each one quickly, which will improve both your timing and performance in critical reasoning questions.

Learn to identify the question types as you complete GMAT critical reasoning practice questions. Identifying the critical reasoning question types as you practice will help you decide how to approach them. To do this, learn the ‘clue words’ and most common question stems for each question type.

Practice with realistic GMAT critical reasoning questions and hone in on your weaknesses. Use the free GMATPrep software or the resources in our GMAT Verbal Practice article to complete a variety of GMAT critical reasoning practice questions. Keep track of how many you get wrong of each question type, and focus on those in your prep time.

Hone in on your weaknesses to maximize the efficacy of your prep!

What’s Next?

If you’d like similar breakdowns of other GMAT question types, check out our top tips for data sufficiency and sentence correction questions.

Want to find out more about the GMAT format overall, as well as get more GMAT critical reasoning tips? Our complete guide to the GMAT format will give you a comprehensive overview of each section.

Our collection of GMAT sample questions is a helpful resource if you’re looking for more ways to practice before the exam (including more critical reasoning GMAT practice questions).

Was this helpful? Sign up for FREE GMAT and MBA guides!

Share this:.

  • Click to share on Twitter (Opens in new window)
  • Click to share on Facebook (Opens in new window)
  • Click to share on Google+ (Opens in new window)

gmat critical thinking sample questions

Author: Laura Dorwart

Laura Dorwart is a Ph.D. student at UC San Diego. She has taught and tutored hundreds of students in standardized testing, literature, and writing. View all posts by Laura Dorwart

Critical Reasoning Practice Test 1

The GMAT Critical Reasoning questions are designed to measure the reasoning skills that you use when you craft arguments, evaluate arguments, and formulate plans of action. It is a challenging section of the test, and you will definitely benefit by working through practice questions. Our free GMAT Critical Reasoning practice test is a great place to start your test prep.

Directions:  Each of these GMAT critical reasoning practice questions are based on a short argument, a set of statements, or a plan of action. For each practice question, select the best answer of the choices given.

  • Which of the following, if true, most strengthens this argument?
  • The adoption of this plan would be most likely to decrease employees' productivity if the employees' job functions required them to __________.
  • Any of the following statements, if introduced as an additional premise, makes the above argument logically correct EXCEPT:
  • Which of the following best suggests that the conclusion suggested by the composer is justified?
  • The conclusion of this argument assumes which of the following?
  • In the given argument, what role do the boldfaced selections play?
  • Which of the following could cause the assurances given by Eatco executives to the sales staff to be untrue?
  • Which of the following, if true, most strongly supports the psychologists’ interpretation of the link between punishment and antisocial behavior?
  • Which of the following, if true, would bolster the McDonald’s manager’s interpretation?
  • All of the following statements, if true for last year, could help to explain the profit growth EXCEPT:

Next Practice Test: Critical Reasoning Practice Test 2>>

More Practice Tests: More Verbal Tests>> Quantitative Tests>> Integrated Reasoning Tests>> Main Menu>>

FB

Critical Reasoning

GMAT verbal critical reasoning

Critical Reasoning Stimuli

Some stimuli contain only facts. But most stimuli contain an argument , in which the author advocates for a particular position based on the facts presented. In these cases, it's important to identify the argument’s primary conclusion . A conclusion can appear anywhere within a stimulus, and may be associated with a keyword such as therefore, thus, hence, demonstrates that, as a result, and conclude. Sometimes the conclusion is a cause-and-effect relationship . Also identify reasons upon which the conclusion is based. These reasons (called premises) may be associated with a keyword such as because, since, assuming, due to, and given that. Finally, arguments typically contain one or more assumptions , unstated facts upon which the conclusion depends.

GMAT Critical Reasoning Question Types

When reading the question stem, identify what the question is asking you to do. According to the GMAT Genius classification system, there are six primary and three applied question types . Each CR question type has certain techniques that can help you choose the correct answer.

  • Weaken – point out a flaw in the argument or make the conclusion less likely to be true.
  • Strengthen – improve the argument or make the conclusion more likely to be true.
  • Assumption – identify an assumption that is absolutely essential for the conclusion to be true.
  • Reasoning – either describe the flawed reasoning used within an argument or describe the argument's structure.
  • Conclusion – draw a conclusion or inference from the facts in the stimulus, or identify what must be true.
  • Explain – reconcile two seemingly contradictory facts that appear in the stimulus.
  • Evaluate – find information that would help you assess the argument's validity.
  • Boldface – describe the role played by two boldfaced portions of the stimulus.
  • Complete the Passage – identify what that would logically complete the ending of the stimulus.

How to Approach GMAT Critical Reasoning

Some companies recommend that you read the question before reading the stimulus. We strongly disagree. There are two disadvantages to reading the question initially . First, you then tend to look for a certain answer while reading the stimulus and may overlook important subtleties. Second, you will often have to read the question a second time, either because you forgot what the question was asking or you didn’t have the context of the stimulus to fully interpret the question. As a result, you waste valuable time.

Instead, critically read and evaluate the stimulus before worrying about the question. After doing so, you can often identify (or at least narrow down) the correct answer without referring back to the stimulus. The best LSAT preparation companies recommend reading the stimulus initially for this reason. That said, identify which approach works best for you . Try 10 CR questions at one go, reading the stimulus first on five questions and then reading the question first on the next five. In our experience, most students recognize, in doing so, that they prefer reading the stimulus first.

After carefully reading the stimulus and question stem, consider the five answer choices. Some companies recommend "pre-thinking" the correct answer first. Pre-thinking is a terrible approach in our opinion. Although this may work on easy-difficulty questions, it rarely works on hard-difficulty questions , which don't list an obvious answer among the answer choices. Furthermore, why waste valuable time anticipating what may be the correct answer instead of just reading the answers presented.

Process of elimination is important when reviewing the answers. Take a first quick pass through all five answers to familiarize yourself with the options and to eliminate answers that are definitely wrong. Do not spend much time evaluating answers in the first pass, because you may identify a subsequent answer as the clear correct choice. Only after the first pass, take a second pass at the worthwhile answers, referring back to the stimulus as necessary to choose the correct answer.

Sample GMAT Critical Reasoning Problem

Let’s try a sample problem. Attempt the problem on your own before viewing the answer and explanation.

There are far fewer kidney donors available than there are people who need kidneys. In the U.S. alone, 95,000 people currently wait on the official kidney transplant list. But two years ago, the last year for which data is available, just 16,500 people received a kidney transplant.

Which of the following statements, if true, most strengthens the author’s claim that there are far fewer kidney donors available than there are people who need kidneys?

Explanation to Problem

The first sentence of the stimulus is the conclusion: there are far fewer kidney donors available than there are people who need kidneys. This is a classic supply-demand imbalance issue. The supply (available kidney donors) cannot meet the demand (number of people on the kidney transplant list). Although we know this to be true in the real world, we must consider whether the author offers an airtight argument.

As premises, the author offers two pieces of data. First, there are currently 95,000 people on the official kidney transplant list. Second, two years ago, there were only 16,500 kidney transplants. Unfortunately, the premises do not offer definite proof of a supply-demand imbalance. Two problems stand out:

The years are different . We do not know how many people were on the official kidney transplant list two years ago, nor do we know how many kidney transplants currently occur. Perhaps only 16,500 people were on the official transplant list two years ago and perhaps 95,000 kidney transplants will take place in the current year. If so, there really is no supply-demand imbalance.

We are told the number of kidney transplants that occurred two years ago, but we do not know how many kidney donors were available. The author is assuming that the number of kidney transplants is a good proxy for the number of available donors, but this doesn’t necessarily have to be the case. Perhaps the number of available donors was equal to the number of people on the transplant list, but far fewer transplants occurred for some other reason, such as bureaucratic snafus.

This is a Strengthen question , as indicated by the words “most strengthens” in the question. For these questions, incorrect answers will either be irrelevant or actually weaken the argument (the opposite of our objective). Our correct answer will most likely focus on fixing one of the flaws that we listed above.

Answer Choices

Prev

gmat critical thinking sample questions

CHIRANJEEV SINGH GMAT Coach

Critical Reasoning- Collection of All Official Guide Questions

The table below lists all the Critical Reasoning questions that have appeared in various editions of Official Guides.

  • You can sort the table according to any column.
  • You can filter questions of a specific type. For example, if you want to look at just Assumption questions, you can type in “Assumption” in the search box right above the table.
  • You can filter questions of a specific difficulty level. For example, if you want to look at just high-difficultly level questions, you can search “high” in the search box.
  • You can filter questions of a specific type and difficulty level. For example, if you want to look at assumption questions of high difficulty level, you can search “assumption high” in the search box.
  • You can choose the number of rows per page by selecting the number above the top-left corner of the table.
  • Right below the bottom-right corner of the table, there are previous and next buttons to access different pages of the table.

Picture of Chiranjeev Singh

Chiranjeev Singh

Join the conversation.

' src=

14 Comments

Can you suggest the best approach for average students to clear quant. I have a difficulty in understanding questions in quant. I feel I migh score less because of the same.

A good approach is to identify exactly where you are going wrong and then work on those areas. These areas could be calculations, certain concepts, handling constraints etc. You can also look at a structured course such as Target Test Prep for your preparation.

– Chiranjeev

Do this list also consists of review book 2020 (vr +qr) ?

Not right now!

This is so helpful, thank you. Will you update this for 2020?

Yes. Working on it.

Updated to include new questions in OG 2020, VR 2020, and Advanced OG guide.

So if we refer to the Verbal Set listed here – Do I need to refer OG 2020 again for Verbal or all questions are covered

All questions are covered here.

This helps a lot. Thank you for putting it all together and making it as a one stop solution

Hi sir. Thank you for doing this. Its immensely helpful 🙂

How can I break the 700 barrier questions, however I try, I get 80 percent wrong in verbal and quant 700 level questions

Thanks for helping us. I wish i had found your website earlier. Now i am at a point where i know all the concepts, but marks still hovering around 700–710. I know tutoring may not help much , but practicing more questions isn’t helping either!!

  • Pingback: Mega-compilation of 4000 Unique Official GMAT Verbal Questions - GMAT with CJ

Leave a comment

Leave a reply cancel reply.

Recent Articles

SC Misconception #2 – Like cannot be used to offer examples

GMAT Aspirant: Are you also measuring yourself the wrong way?

Countable and Uncountable nouns

Analysis of my ESR (Enhanced Score Report)

SC Misconception #6 – Which always presents non-essential information

Don’t swim against the tide of GMAT

Recent Solutions

Asthma, a chronic breathing disorder, is significantly more common

The 32 species that make up the dolphin family are, the stars, some of them at tremendous speeds, a surge in new home sales and a drop in weekly unemployment, recently physicians have determined that stomach ulcers, in california, a lack of genetic variation in the argentine, deserve your target score.

Learn GMAT the right way – through common sense and logic. There’s no shortcut to a well-deserved success!

Infomation Links

Connect with me!

© 2021 All rights Reserved.

Discover more from GMAT with CJ

Subscribe now to keep reading and get access to the full archive.

Type your email…

Continue reading

ME Podgorica

Recently viewed courses

Recently viewed.

Find Your Dream School

This site uses various technologies, as described in our Privacy Policy, for personalization, measuring website use/performance, and targeted advertising, which may include storing and sharing information about your site visit with third parties. By continuing to use this website you consent to our Privacy Policy and Terms of Use .

   COVID-19 Update: To help students through this crisis, The Princeton Review will continue our "Enroll with Confidence" refund policies. For full details, please click here.

GMAT Sample Questions

Hidden laptop with hands typing, displaying a test diagram above the keyboard

Want a preview of the question types you'll face on the GMAT? Try your hand at the GMAT practice questions below. Then, check your answers against our in-depth explanations to see how you did.

We pulled these GMAT sample questions from our book Cracking the GMAT and from our test prep course materials. For more GMAT practice, take a full-length practice test with us held under the same testing conditions as the real thing. Find out how you'd score, and get  a personalized score report from us that shows your strengths and weaknesses.

  • GMAT Verbal Questions 
  • GMAT Math Questions
  • GMAT Integrated Reasoning Questions 
  • Essay Prompt 

Below you'll find sample GMAT questions covering the three question types you'll encounter on the Verbal section: Sentence Correction , Critical Reasoning, and Reading Comprehension.

GMAT Sentence Correction Questions

1. In order to better differentiate its product from generic brands, the cereal company first hired a marketing firm that specializes in creating campaigns to build brand awareness and then retools its factory to produce a variety of different shapes of cereal. (A) then retools its factory to produce a variety of different shapes of cereal (B) retools its factory to produce a variety of different shapes of cereal (C) then retooled its factory to produce a variety of different shapes of cereal (D) then will retool its factory to produce a variety of different shapes of cereal (E) then produces a variety of different shapes of cereal through retooling its factory

Answer: (C) The actions of the cereal company are not in parallel form. First the company hired then it retools . Eliminate choice (A). Choice (B) still has the same error. Choice (D) changes the verb form incorrectly to the future tense. Choice (E) rewrites the sentence but retains the error.

[+] See the Answer

2. Believed to be one of the first widely read female authors of the Western world, Christine de Pizan's masterwork The Book of the City of the Ladies , was written in 1405 and is a history of the Western world from the woman's point-of-view. (A) Believed to be one of the first widely read female authors of the Western world (B) Written by one of the first widely read female authors of the Western world (C) One of the first widely read female authors of the Western world, as some believe (D) Written by what some believe as one of the first widely read female authors of the Western world (E) Believed by some as one of the first works by a widely read female author in the Western world

Answer: (B) As written, this sentence has a misplaced modifier error: the book, The Book of the City of the Ladies isn't believed by anyone to be an author— Christine de Pizan is. Choices (A) and (C) repeat that error and can be eliminated. Choices (B) and (D) both change the introductory phrase to clearly refer to a written work, but choice (D) uses the incorrect idiom believe as instead of the correct form, believe to be . Choice (E) repeats that idiom error.

GMAT Critical Reasoning Questions

1. One food writer wrote that reducing the amount of animal products in one's diet can contribute to better health and well-being. Based on this claim, some people are completely eliminating meat from their diets in order to be healthier. The argument above relies on which of the following assumptions?

Answer: (B) The argument states that some people are eliminating meat from their diets because reducing the amount of animal products in one's diet can lead to better health. Meat is only one type of animal product, however. The argument assumes that by eliminating meat, the people are reducing the total amount of animal products in their diets. Choice (A) addresses increasing the amount of vegetables and grains, but the argument just deals with animal products. Choice (B) correctly addresses the people who are eliminating meat and states that those people are not increasing their consumption of dairy, which is another instance of using animal products. Thus, these people are actually reducing the amount of animal products in their diets. Choice (C) addresses most food writers, who are irrelevant to this argument. Choice (D) addresses health lifestyles, which are irrelevant to this particular argument. Choice (E) addresses the reasons behind not eating animal products, which is irrelevant to the argument.

2. Studies reveal that a daily exercise regimen helps stroke survivors regain dexterity in their extremities. Being given an exercise routine and having a consultation with a doctor about the exercise routine have been shown to be effective mechanisms to get patients to exercise daily. From the above information, which of the following statements can be reasonably inferred? (A) A stroke survivor that is given a detailed exercise plan and consults her physician about the plan will regain full dexterity in her extremities. (B) If a stroke survivor is not given an exercise plan and does not consult with a doctor, she will not regain dexterity in her extremities. (C) Stroke survivors who are given an exercise routine and consult with a doctor about that routine will sometimes regain dexterity in their extremities. (D) Being given an exercise routine and having a consultation with a doctor about the routine is the best way to help a stroke survivor regain dexterity in their extremities. (E) Only being given an exercise routine is necessary to regenerate dexterity in the extremities of seniors who have suffered a stroke.

Answer: (C) This is an inference question, so evaluate the passage and then look for an answer choice that can be reasonably inferred from the information. The passage states that a daily exercise regimen helps stroke survivors regain dexterity in their extremities and that survivors who are given an exercise routine and who have a consultation with a doctor about the routine have been shown to be effective at getting patients to exercise daily . So it can be inferred that if a survivor is given a routine and consults with a doctor, they are more likely to exercise daily, which will help them regain dexterity. Choice (A) is an example of extreme language. The phrasing will regain full dexterity is not promised in the information in the passage, as the passage only states that a routine and consultations may help a survivor exercise more. Eliminate (A). Choice (B) is also an example of extreme language. There is no way to discern from the information provided that a strong survivor would not regain dexterity without an exercise routine and a consultation, so eliminate (B). Choice (C) is a reasonable inference to make from the information in the passage so keep (C). Choice (D) also contains the extreme language best way . The information does not compare this method with any other method so eliminate (D). Choice (E) is recycled language and does not address consulting with a doctor so eliminate (E). The correct answer is (C).

GMAT Reading Comprehension Questions

Although oft-maligned in modern culture, the pigeon once stood not only for speed and reliability but also for grace and beauty. Darwin himself became a pigeon fancier after beginning to work with the humble Columbia livia , discovering them to be more fascinating than he had formerly believed. During the Victorian age, in fact, raising show pigeons was a popular hobby, with new breeds continuously arising as amateur (and not-so-amateur) ornithologists crossed animals in the hopes of creating ever more fantastic creatures. One of the most sought-after varieties was known as the Almond Tumbler, a name presumably derived from the color of the birds combined with the distinctive flight style. Over the course of many generations, this bird was so manipulated as to have a beak so small as to prevent the adult birds from feeding their offspring. And yet, it was wildly popular, drawing high prices at auctions and high prizes at competitions. How then did an animal once so well-loved come to be so loathed? As recently as World War II, the military used pigeons to carry messages but today, many people would kick a pigeon before they would feed one. Perhaps it is just a problem of population density - a lack of esteem for that which is ubiquitous. Pigeons have become our constant urban companions and, as such, have been transformed from symbols of peace, plenty, and prosperity, to representatives of disease and decay.

1. The primary purpose of this passage is to (A) convince the reader of the nobility of the pigeon, based on its history as a symbol of virtue (B) dissuade the reader from mistreating a once-majestic animal that has fallen from favor (C) rebut claims that the pigeon carries disease any more frequently than do other domestic animals (D) promote a renewal of pigeon fancying and a resurgence of breeds such as the Almond Tumbler (E) suggest that there might be more to the story of some urban wildlife than is commonly known

Answer: (E) The passage gives a brief description of the pigeon's place in recent human history and then goes on to contrast that with modern perspectives of the birds. Choice (A) goes too far—the author doesn't give any indication of believing the pigeon to be noble. Choice (B) focuses too specifically on a side comment in the second paragraph. Choice (C) also focuses too specifically on a side comments—the passage is not primarily about disease. Choice (D) is too strong—the passage isn't really promoting any specific action. Choice (E) remains neutral and informational, as does the passage.

2. The case of the Almond Tumbler is most analogous to which of the following? (A) a strain of wheat that can be grown in plentiful quantities but loses much of its nutritional value in the process (B) Arabian horses that are able to run at phenomenal speeds due to centuries of careful breeding designed to enhance those physical attributes (C) vitamins that were purported to provide all of the necessary nutrients but have since been found not to be very effective (D) the dachshund, a popular breed of dog that is nonetheless prone to severe back problems, due to weaknesses exacerbated by targeted breeding (E) the wild rock doves that are most commonly found nesting in the faces of cliffs far from human habitation

Answer: (D) The Almond Tumbler is described as a breed of pigeon that was very popular during the Victorian era. The passage also mentions that the selective breeding used to create that particular kind of bird also led to tiny beaks that kept parent birds from feeding their babies. Therefore, the best analogy would be another animal that is popular even though it has problems due to its design. Choice (A) is incorrect because it leaves out the aspect of popularity. Choice (B) is only positive and you need something that's also negative. Choice (C) is not about something that has been bred for a specific purpose, nor does it deal with popularity. Choice (D) correctly refers to a popular animal with a common health problem. Choice (E) does not refer to pigeons that have been bred by humans.

3. The passage suggests that (A) pigeons were once known for flying with celerity (B) the Almond Tumbler was the most beautiful breed of pigeon (C) Darwin was infatuated with his fancy pigeons (D) modern pigeons are dirtier than the fancy pigeons of yore (E) only scientists should breed new kinds of animals

Answer: (A) For a question this open-ended, it's usually best to check each of the answers against the passage. Choice (A) appears to match the opening line of the passage, which states that the pigeon once stood not only for speed and reliability. Choice (B) goes too far—although many Victorians seems to have loved the Tumbler, there's no evidence that it was definitively the most beautiful. Choice (C) also goes too far—the passage mentions that Darwin was fascinated by his pigeons, not that he was infatuated. Choice (D) draws an incorrect assumption—the passage comments that the common opinion has changed, not the pigeon itself. Choice (E) is not supported by the passage, which states that amateurs, as well as trained individuals, bred pigeons.

Below you'll find GMAT sample questions covering the two question types you'll encounter on the Quantitative section: Problem Solving and Data Sufficiency.

Problem Solving Questions

1. A certain company sells tea in loose leaf and bagged form, and in five flavors: Darjeeling, earl grey, chamomile, peppermint, and orange pekoe. The company packages the tea in boxes that contain either 8 ounces of tea of the same flavor and the same form, or 8 ounces of tea of 4 different flavors and the same form. If the order in which the flavors are packed does not matter, how many different types of packages are possible? (A) 12 (B) 15 (C) 20 (D) 25 (E) 30

Answer: (C) Begin by figuring out how many different ways you can package the tea in boxes that contains 8 ounces of tea, all of the same flavor. There are five flavors, each flavor can come in either loose leaf or bagged form, so 5 flavors x 2 forms = 10 different ways to package the tea in boxes that contain only one flavor each. Now find the number of different ways to package 4 different flavors of the same form per box. In this case, you must choose 4 of 5 possible flavors, and order does not matter, so the formula is 5 x 4 x 3 x 2 ⁄ 4 x 3 x 2 x 1 = 5 different ways to combine the 4 flavors. Each combination can come in either loose leaf for bagged form, so you have 2 different forms x 5 different combinations = 10 total possible ways to combine the 4 flavors in either bagged or loose-leaf form. Thus, the total number of combinations is 10 + 10 = 20 total combinations. The answer is choice (C).

2. Karen sold her house at a loss of 25 percent of the price that she originally paid for the house, and then bought another house at a price of 30 percent less than the price she originally paid for her first house. If she sold the first house for $225,000, what was her net gain, in dollars, for the two transactions? (A) $15,000 (B) $25,000 (C) $60,000 (D) $75,000 (E) $90,000

Answer: (A) If Karen sold her first house for $225,000 and at a loss of 25 percent, then 25 percent of the original price equals $225,000. 75 ⁄ 100 x = 225,000, so x, or the price she originally paid, equals $300,000. Thus, Karen lost $75,000 on the sale of her first house. If she bought a second house for a price of 30 percent less than $300,000, then the second house cost $210,000, so she gained $90,0000. $90,000 - $75,000 = $15,000, so the answer is choice (A).

Sample Data Sufficiency Questions

1. In a certain company, at least 200 people own manual transmission vehicles. If 12 percent of the people who own manual transmission vehicles also own automatic transmission vehicles, do more people own automatic transmission vehicles than own manual transmission vehicles? (1) 5 percent of the people who own an automatic transmissions vehicle also own a manual transmission vehicle. (2) 15 people own both an automatic transmission vehicle and a manual transmission vehicle. (A) Statement (1) ALONE is sufficient, but statement (2) alone is not sufficient. (B) Statement (2) ALONE is sufficient, but statement (1) alone is not sufficient. (C) BOTH statements TOGETHER are sufficient, but NEITHER statement ALONE is sufficient. (D) EACH Statement ALONE is sufficient. (E) Statements (1) and (2) TOGETHER are NOT sufficient to answer the question asked, and additional data are needed.

Answer: (A) According to statement (1), 5 percent of the people who own an automatic transmission vehicle also own a manual transmission vehicle. The question also indicates that 12 percent of the people who own a manual transmission vehicle also own an automatic transmission vehicle. Both figures relate to the total number who own both, so that means that 5 percent of the automatic transmission owners = 12 percent of the manual transmission owners. The overlap in ownership makes up a smaller percent of those who own automatic transmission vehicles, so there must be more people who own automatic transmission vehicles. Statement (1) is sufficient, so you can eliminate choices (B), (C), and (E). Statement (2) indicates that 15 people own both an automatic transmission vehicle and a manual transmission vehicle, so you know that 12 percent of the people who own a manual transmission is equal to 15 people. 12 ⁄ 100 = 15, so x = 125. Thus, there are 125 people who own a manual transmission vehicle. However, you have no further information to allow you to calculate the number of people who own automatic transmission vehicles, so statement (2) is insufficient. The answer is choice (A).

2. What is the value of x ⁄ 2 ? (1) x is 1 ⁄ 5 less than 9 ⁄ 10 (2) x is between 2 ⁄ 5 and 4 ⁄ 5 (A) Statement (1) ALONE is sufficient, but statement (2) alone is not sufficient. (B) Statement (2) ALONE is sufficient, but statement (1) alone is not sufficient. (C) BOTH statements TOGETHER are sufficient, but NEITHER statement ALONE is sufficient. (D) EACH Statement ALONE is sufficient. (E) Statements (1) and (2) TOGETHER are NOT sufficient to answer the question asked, and additional data are needed.

Answer: (A) Statement (1) allows you to find the value of x, so you can answer the question. (If x is 1 ⁄ 5 less than 9 ⁄ 10 , then 9 ⁄ 10 - 1 ⁄ 5 = x. 1 ⁄ 5 = 2 ⁄ 10 , so x equals 9 ⁄ 10 - 2 ⁄ 10 = 7 ⁄ 10 . If x equals 7 ⁄ 10 , then x ⁄ 2 = 7 ⁄ 10 divided by 2, or 7 ⁄ 20 .) Statement (1) is sufficient, so eliminate choices (B), (C), and (E). According to statement (2), x is between 2 ⁄ 5 and 4 ⁄ 5 . That means that one possible value for x is 3 ⁄ 5 , but another possible value is 7 ⁄ 10 . Statement (2) is insufficient, so the answer is choice (A).

Below you'll find examples of how you'll be asked to use a chart, graph, or table to answer questions on the Integrated Reasoning section.

Sample Integrated Reasoning Questions

Item 1: Andre is buying gifts for his office staff. He wants to spend exactly $280 and he can buy either sweatshirts, which cost $22, or baseball caps, which cost $26. In the table below, choose the number of sweatshirts and the number of baseball caps that Andre should buy.

Answer: Sweatshirts, 8; Baseball caps 4 To solve this question, systematically test out the answer choices. The equation you need to solve is 22s + 26h = 280, in which both s and h are integers and s represents the number of sweatshirts and h represents the number of baseball caps. So, start with plugging in 4 for sweatshirts and see if the number of baseball caps is an integer. 22(4) +26h = 280 h = 7.38 Since the number of baseball caps is not an integer, Andre could not have bought 4 sweatshirts. Keep trying more sweatshirts one by one until you find an answer that will you an integer value for baseball caps. 8 sweatshirts will give you 4 baseball caps.

GMAT sample question

Question 2-1 The ratio of the U.S. population in 2000 to the U.S. population in 1900 is closest to __. (A) 1 to 4 (B) 2 to 7 (C) 2 to 1 (D) 3 to 1 (E) 11 to 3

Answer: (E, 11 to 3) According to the graph, the U.S. population in 2000 was a little bit more than 275 million, and the U.S. population in 1900 was a little over 75 million. Since the question asks what the ratio is "closest to," these numbers are good enough to approximate. 275 to 75 can be reduced by 5 to get 55 to 15, which can be reduced by 5 again to get 11 to 3. Alternatively, you could reduce 275 to 75 by 25 to get this same ratio.

Question 2-2 The U.S. population in 1950 was approximately __ of the U.S. population in 1850. (A) 800% (B) 600% (C) 200% (D) 85% (E) 15%

Answer: (B, 600%) The question asks what percent the U.S. population in 1950 is of the U.S. population in 1850. To get this you need to calculate population 1950 ⁄ population 1850 x 100. Since the U.S. population in 1950 is higher, you want something that is greater than 100%. Eliminate 85% and 15%. Since the sentence says "approximate" and also since the remaining answer choices are not close to each other, you can estimate the values. According to the chart, the population in 1950 was about 150 million and the population in 1850 was about 25 million. Therefore, you need to calculate 150 ⁄ 25 x 100 = 6 x 100 = 600%.

Question 2-3 The U.S. population increased by approximately __ from 1900 to 1950. (A) 25% (B) 33% (C) 50% (D) 100% (E) 200%

Answer: (D, 100%) To get percent increase, you need to use the formula difference ⁄ original x 100. The population in 1900 was about 75 million, and the population in 1950 was about 150 million. The difference between the two figures is 75 million. Therefore, the percent increase is 75 ⁄ 75 x 100 = 100%.

Below you'll find a sample Analytical Writing Assessment (AWA) question. On the GMAT you'll have 30 minutes to write a critique of the argument.

Analysis of an Argument

The following appeared as part of a medical advertisement in a magazine.

A new medical test that allows the early detection of a particular disease will prevent the deaths of people all over the world who would otherwise die from the disease. The test has been extremely effective in allowing doctors to diagnose the disease six months to a year before it would have been spotted by conventional means.

Discuss how logically convincing you find this argument. In explaining your point of view, be sure to evaluate the line of reasoning and the use of evidence in the argument. For example, it may be necessary to consider what questionable assumptions underlie the thinking and what other explanations or counterexamples might weaken the arguments conclusion. You can also discuss what kind of evidence would strengthen or refute the argument, what changes in the argument would make it more logically persuasive, and what, if anything, would enable you to better evaluate its conclusion.

How will you score?

Take a GMAT practice test with us under the same conditions as the real thing. You'll get a personalized score report highlighting your strengths and areas of improvement.

START A FREE PRACTICE TEST

  • GMAT  

Featured Business Schools For You

Find MBA Programs Matched to Your Interests

Explore our featured business schools to find those that are looking for students like you.

Best Online MBA seal

Top Online MBA Programs

On a mission to increase your salary? Our Top 50 Online MBA ranking is based on academics, career outcomes, tech platforms, and more.

Best Career Prospects

Best Career Prospects

Find out which schools have the best track records for getting students jobs—and the highest starting salaries.

Top Entrepreneurship 2023 seal

Top Schools for Entrepreneurship

Ready to build your own business from the ground up? Check out these 50 graduate programs.

gmat critical thinking sample questions

Free MCAT Practice Test

I already know my score.

gmat critical thinking sample questions

MCAT Self-Paced 14-Day Free Trial

gmat critical thinking sample questions

Enrollment Advisor

1-800-2REVIEW (800-273-8439) ext. 1

1-877-LEARN-30

Mon-Fri 9AM-10PM ET

Sat-Sun 9AM-8PM ET

Student Support

1-800-2REVIEW (800-273-8439) ext. 2

Mon-Fri 9AM-9PM ET

Sat-Sun 8:30AM-5PM ET

Partnerships

  • Teach or Tutor for Us

College Readiness

International

Advertising

Affiliate/Other

  • Enrollment Terms & Conditions
  • Accessibility
  • Cigna Medical Transparency in Coverage

Register Book

Local Offices: Mon-Fri 9AM-6PM

  • SAT Subject Tests

Academic Subjects

  • Social Studies

Find the Right College

  • College Rankings
  • College Advice
  • Applying to College
  • Financial Aid

School & District Partnerships

  • Professional Development
  • Advice Articles
  • Private Tutoring
  • Mobile Apps
  • Local Offices
  • International Offices
  • Work for Us
  • Affiliate Program
  • Partner with Us
  • Advertise with Us
  • International Partnerships
  • Our Guarantees
  • Accessibility – Canada

Privacy Policy | CA Privacy Notice | Do Not Sell or Share My Personal Information | Your Opt-Out Rights | Terms of Use | Site Map

©2024 TPR Education IP Holdings, LLC. All Rights Reserved. The Princeton Review is not affiliated with Princeton University

TPR Education, LLC (doing business as “The Princeton Review”) is controlled by Primavera Holdings Limited, a firm owned by Chinese nationals with a principal place of business in Hong Kong, China.

  • Skip to content

Free GMAT Test Questions

Welcome to our Question Bank!

You have not answered any question so far. There are 50 free practice questions in our database in total, which you can answer and will improve your skills.

You can answer all questions in a row (click on "All Questions") or only all questions of a particular section (click on that Section) or a single selected question (click on that Question).

All Questions

Quantitative reasoning - problem solving, quantitative reasoning - data sufficiency, verbal reasoning - critical reasoning, verbal reasoning - reading comprehension, verbal reasoning - sentence correction.

GMAT is a registered trademark of the Graduate Management Admission Council (GMAC), which is unaffiliated with and does not endorse this website.

Sentence Correction Secrets

Image

Wouldn't it be great to know the average GMAT score of the business school of your dreams? You've studied hard for the GMAT, but are you fully sure of what ...

Adaptive Testing Explained

Image

Getting the Most Out of Your Prep Class

gmat critical thinking sample questions

  • Exam Prep >
  • Prepare for Business School >
  • Business School & Careers >
  • Explore Programs >
  • Connect with Schools >
  • How to Apply >
  • Help Center >

Every journey needs a plan. Use our Career Guide to get where you want to be. 

  • About the Exam
  • Register for the Exam
  • Plan for Exam Day
  • Prep for the Exam
  • About the Executive Assessment
  • Register for the Executive Assessment
  • Plan for Assessment Day
  • Prepare for the Assessment
  • NMAT by GMAC
  • Shop GMAT Focus Official Prep
  • About GMAT Focus Official Prep
  • Prep Strategies
  • Personalized Prep Plan
  • GMAT Focus Mini Quiz
  • Executive Assessment Exam Prep
  • NMAT by GMAC Exam Prep

Prepare For Business School

  • Business Fundamentals
  • Skills Insight

Business School & Careers

  • Why Business School
  • Student Experience
  • Business Internships
  • B-School Go
  • Quiz: Are You Leadership Material?
  • MBA Return on Investment (ROI) Calculator
  • Estimate Your Salary
  • Success Stories
  • Diversity and Inclusion
  • Women in Business

Explore Programs

  • Top Business School Programs
  • Quiz: Which Post Graduate Program is Right for You?
  • Quiz: Find the Best Program for Your Personality
  • Business School Rankings
  • Business Master's Programs
  • MBA Programs
  • Study Destinations
  • Find Programs Near Me
  • Find MBA Programs
  • Find Master's Programs
  • Find Executive Programs
  • Find Online Programs

Connect with Schools

  • About GradSelect
  • Create a GradSelect Profile
  • Prep Yourself for B-School
  • Quiz: Can You Network Like An MBA?
  • Events Calendar
  • School Events
  • GMAC Tours Events
  • In-Person Events
  • Online Events

How to Apply

  • Apply to Programs
  • The Value of Assessments
  • Admissions Essays
  • Letters of Recommendation
  • Admissions Interviews
  • Scholarships and Financing
  • Quiz: What's Your Ideal Learning Style?

Help Center

  • Register for the GMAT
  • Create Account

Sample Questions

Quick links.

Before you register  for the GMAT Focus Edition, get a preview of the types of questions you’ll encounter on the exam. 

Remember: You can learn more about section and question type at Exam Content .

Sample Questions by Section

Quantitative reasoning.

Problem Solving Question Directions: Solve the problem and indicate the best of the answer choices given. Question: If a certain wheel turns at a constant rate of x revolutions per minute, how many revolutions will the wheel make in k seconds? (A) 60 kx (B)  kx (C) x ÷ k (D) x ÷(60 k ) (E) kx ÷60 Answer:  (E)

Verbal Reasoning

Reading Comprehension Question Directions: The questions in this group are based on the content of a passage. After reading the passage, choose the best answer to each question. Answer all questions following the passage on the basis of what is stated or implied in the passage. Question: Schools expect textbooks to be a valuable source of information for students. My research suggests, however, that textbooks that address the place of Native Americans within the history of the United States distort history to suit a particular cultural value system. In some textbooks, for example, settlers are pictured as more humane, complex, skillful, and wise than Native Americans. In essence, textbooks stereotype and depreciate the numerous Native American cultures while reinforcing the attitude that the European conquest of the New World denotes the superiority of European cultures. Although textbooks evaluate Native American architecture, political systems, and homemaking, I contend that they do it from an ethnocentric, European perspective without recognizing that other perspectives are possible. One argument against my contention asserts that, by nature, textbooks are culturally biased and that I am simply underestimating children's ability to see through these biases. Some researchers even claim that by the time students are in high school, they know they cannot take textbooks literally. Yet substantial evidence exists to the contrary. Two researchers, for example, have conducted studies that suggest that children's attitudes about particular cultures are strongly influenced by the textbooks used in schools. Given this, an ongoing, careful review of how school textbooks depict Native Americans is certainly warranted. Which of the following would most logically be the topic of the paragraph immediately following the passage? (A) Specific ways to evaluate the biases of United States history textbooks (B) The centrality of the teacher's role in United States history courses (C) Nontraditional methods of teaching United States history (D) The contributions of European immigrants to the development of the United States (E) Ways in which parents influence children's political attitudes Answer:  (A)

Critical Reasoning Question Directions: For this question, select the best of the answer choices given. Question: Mall Owner: Our mall’s occupancy rate is so low that we are barely making a profit. We cannot raise rents because of an unacceptably high risk of losing established tenants. On the other hand, a mall that is fully occupied costs about as much to run as one in which a rental space here and a rental space there stands empty. Clearly, therefore, to increase profits we must sign up new tenants. Which of the following, if true, most seriously weakens the argument? (A) The mall’s operating costs could be cut by consolidating currently rented spaces in such a way that an entire wing of the mall could be closed up. (B) The mall is located in a geographic area in which costs incurred for air-conditioning in the hot summers exceed those incurred for heating in the mild winters by a wider margin. (C) The mall’s occupancy rate, though relatively low, has been relatively stable for several years. (D) The mall lost tenants as a result of each of the two major rent increases that have occurred there. (E) None of the mall’s established tenants is likely to need additional floor space there in the foreseeable future. Answer:  (A)

Data Insights

Data Sufficiency Question Directions: This data sufficiency problem consists of a question and two statements, labeled (1) and (2), in which certain data are given. You have to decide whether the data given in the statements are sufficient for answering the question. Using the data given in the statements, plus your knowledge of mathematics and everyday facts (such as the number of days in July or the meaning of the word counterclockwise), you must indicate whether:

  • Statement (1) ALONE is sufficient, but statement (2) alone is not sufficient to answer the question asked.
  • Statement (2) ALONE is sufficient, but statement (1) alone is not sufficient to answer the question asked.
  • BOTH statements (1) and (2) TOGETHER are sufficient to answer the question asked, but NEITHER statement ALONE is sufficient to answer the question asked.
  • EACH statement ALONE is sufficient to answer the question asked.
  • Statements (1) and (2) TOGETHER are NOT sufficient to answer the question asked, and additional data specific to the problem are needed.

Question: What is the monthly rent for a certain apartment? (1) The monthly rent per person for 4 people to share the rent for the apartment is $375. (2) The monthly rent per person for 4 people to share the rent of the apartment is $125 less than the monthly rent per person for 3 people to share the rent. (A) Statement (1) ALONE is sufficient, but statement (2) alone is not sufficient. (B) Statement (2) ALONE is sufficient, but statement (1) alone is not sufficient. (C) BOTH statements TOGETHER are sufficient, but NEITHER statement ALONE is sufficient. (D) EACH statement ALONE is sufficient. (E) Statements (1) and (2) TOGETHER are NOT sufficient. Answer: (D) For the other four question types, click for samples of each question type to open up in an interactive pop-up window. Please note: these sample questions are built to simulate the actual test interface, and therefore, are not optimized for mobile devices.

  • Multi-Source Reasoning
  • Table Analysis
  • Graphics Interpretation
  • Two-Part Analysis

The Magoosh logo.

How to Tackle Critical Reading Assumption Questions

Students debating assumption questions and GMAT critical reasoning tips - image by Magoosh

Assumption questions ask you to find the unstated link between a question’s premise and its conclusion. Assumptions are crucial in understanding and refuting arguments, so they play a large role in two major Critical Reasoning question types . In this post, we’ll cover GMAT Critical Reasoning tips and practice questions to help you tackle assumption questions.

How to Tackle Critical Reasoning Assumption Questions

Luckily, arguments on GMAT Critical Reasoning questions are relatively formulaic, so let’s go over the basics first:

  • A premise is the starting point of the argument.
  • The conclusion is what the author wants you to believe by the end of the argument.
  • The assumption is the missing link between the premise and conclusion. Think of it like the linchpin holding the whole thing together. You can strengthen an argument by validating its assumption, or weaken the argument by denying the assumption.

Assumption questions will usually ask you, “Which would most strengthen the argument?” or “Which of the following would most weaken the argument?” (the latter is one of the most common on Critical Reasoning).

Make Your Assumption a General Statement

This is a crucial point to remember: assumptions are most often general statements , not specific statements. When you identify the assumption, you can omit any specific people, places, or items mentioned.

If my premise is “Fred has quality A,” and my conclusion is “Therefore, Fred has quality B,” Fred is a specific person that we can omit (sorry, Fred). The assumption would be something like “most/all folks who have quality A also have quality B.”

Identify the Assumption

Isolating an assumption is an important skill and one of our favorite GMAT Critical Reasoning tips. Let’s try it with this argument:

  • The premise is “Hawaii is a place with beautiful scenery.” (We can safely assume that at least 99 out of a hundred people would agree with that!) Hawaii is the specific, so you can omit that—the final premise has to do with a “place with beautiful scenery.”
  • The conclusion is “trouble concentrating.”
  • The assumption must provide a link. If we put those together with a strong logical connection, we get this assumption: “People in places with beautiful scenery generally have trouble concentrating.” Even though it’s a little absurd, that’s a possible way to state the assumption!

It would most strengthen this argument if one could somehow provide data or evidence supporting this assumption. This argument would be weakened if we could cite data or evidence that directly contradicts the assumption.

Now, consider an argument you’re more likely to see on the GMAT:

  • If we drop the specifics, the premise is about increasing spending on advertising, and the conclusion is: more new customers. An assumption would link these.
  • A very broad assumption: “Companies that increase what they spend on advertising generally see an increase in new customers.”
  • A slightly more specific assumption: “When companies in the steel industry increase advertising, this generally results in more new customers.”

This is a relatively poor argument, and if we were asked for a statement to weaken it, the best choice would be something that zeroed in on the assumption. For example, something like Studies of companies in the steel industry show little correlation between advertising dollars and new customers strikes right at the center of the argument.

Use the Negation Test to Verify the Assumption

If you want to verify that your assumption is really the correct one, you can use the Negation Test —put simply, try negating the statement and seeing if the conclusion is still true. If you haven’t tried the Negation Test yet (another of our key GMAT Critical Reasoning tips!), then I would definitely recommend checking out our post and studying this powerful technique for isolating assumptions of arguments.

Practice Questions and Explanations

1. Which of the following is an assumption that supports drawing the conclusion above from the reasons given for that conclusion?

Click here for the answer and video explanation!

2. Which of the following is an assumption that supports drawing the conclusion above from the reasons given for that conclusion?

3. Which one of the following is an assumption on which the conclusion depends?

If folks with Laestrygonian Disease cannot assimilate the Vitamin C in the rice, then it won’t help them, and eating the fortified rice will not provide them any particular benefit. If we negate this option, it shatters the argument. This is a true assumption.

(A) This may be true, although I am skeptical that any human-made improved food would be better than the fruits designed by Nature! Regardless, whether this is true or not does not have any bearing on how helpful the fortified rice will be for the folks with Laestrygonian Disease. This option is incorrect.

(B) This is intriguing. Let’s negate this. Suppose it were the exact same problem, say, the exact same missing enzyme, that made it impossible to digest both fruit and vitamin supplements. Then what? Would that mean they also couldn’t digest the fortified rice, or get the vitamin C they need from it? We cannot say. It’s conceivable that the argument could still work, so negating this does not destroy the argument. This is not an assumption.

(D) Let’s negate this. Suppose the fortified rice benefits everyone—even the no-carbs fanatic who hasn’t touched carbs in a decade: even when this person breaks his carb-fast and has the fortified rice, he has benefit from it. What then? Whether these other people benefit or not from the fortified rice has no bearing on whether it helps the folks with Laestrygonian Disease. This choice is incorrect.

(E) Let’s negate this. Suppose we can infused dozens of other vitamins and minerals into the rice, all with high nutritional yield. That would only be good for the folks with Laestrygonian Disease—the more vitamins, the better! It certainly would not impact whether these folks derived any benefit from the vitamin C in the rice. This choice is incorrect.

Final Thoughts

Assumption questions will require you to read closely, but with practice you can identify the missing link. For more GMAT Critical Reasoning tips, check out our introduction to the CR section , then test your knowledge further with a GMAT Verbal diagnostic test .

Mike MᶜGarry

Mike served as a GMAT Expert at Magoosh, helping create hundreds of lesson videos and practice questions to help guide GMAT students to success. He was also featured as “member of the month” for over two years at GMAT Club . Mike holds an A.B. in Physics (graduating magna cum laude ) and an M.T.S. in Religions of the World, both from Harvard. Beyond standardized testing, Mike has over 20 years of both private and public high school teaching experience specializing in math and physics. In his free time, Mike likes smashing foosballs into orbit, and despite having no obvious cranial deficiency, he insists on rooting for the NY Mets. Learn more about the GMAT through Mike’s Youtube video explanations and resources like What is a Good GMAT Score? and the GMAT Diagnostic Test .

View all posts

More from Magoosh

Student holding a pencil and thinking

Leave a Comment

Please leave any questions or suggestions in the comments, we try our best to respond within a few days! Your email address will not be published.

Leave a Reply Cancel reply

Your email address will not be published. Required fields are marked *

GMAT Verbal : Inference Critical Reasoning

Study concepts, example questions & explanations for gmat verbal, all gmat verbal resources, example questions, example question #1 : inference critical reasoning.

Barry’s Barbecue is a restaurant chain that advertises itself as a safe place for diners with food allergies to eat. At Barry’s, whenever a diner books a reservation and mentions a food allergy, the kitchen staff is prohibited from preparing multiple dishes on the same grill. This ensures that there is no cross-contamination between dishes, but also can result in longer wait times as fewer meals can be prepared than would be the case under normal circumstances.

Which of the following is best supported by the information above?

Barry’s will not make special kitchen accommodations for diners who do not make a reservation.

Limiting cross-contamination from multiple dishes on the same grill is the most effective way to avoid issues for diners with food allergies.

The kitchen staff at Barry’s sometimes prepares multiple dishes on the same grill.

Not all restaurants follow food allergy precautions to avoid cross-contamination between multiple dishes.

Diners with food allergies are generally willing to be patient with longer wait times in order to avoid cross-contamination between dishes.

With Inference questions, the correct answer has to fit the "must be true" standard, meaning that it has to be proven based on the passage; incorrect answers "could be true" but are not necessarily true based only on the information in the passage.

Here choice "The kitchen staff at Barry’s sometimes prepares multiple dishes on the same grill." fits that standard largely because of the phrase "under normal circumstances" at the end of the stimulus. If the prohibition on preparing multiple dishes on the same grill is different from "under normal circumstances," then it must be true that "sometimes" (note: "sometimes" is a very low bar to clear for proof) multiple dishes are prepared on the same grill. Choice "The kitchen staff at Barry’s sometimes prepares multiple dishes on the same grill." is therefore correct.

In contrast, notice the strong language within choice "Barry’s will not make special kitchen accommodations for diners who do not make a reservation.", that the restaurant categorically  will not  make kitchen accommodations (of any type) if a diner does not make a reservation. From the stimulus you know of one particular accommodation that will be made under a reservation, but you cannot conclude that there are no other possible accommodations, or that the restaurant wouldn't try to make that accommodation if someone were to arrive without a reservation.

Choice "Not all restaurants follow food allergy precautions to avoid cross-contamination between multiple dishes." could possibly be true ("not all" is another low bar of proof) but as this stimulus only tells you about one particular accommodation that one particular restaurant makes, you just do not have evidence to support this. (Note that while "not all" is a low bar, "food allergy precautions" is fairly broad: if every restaurant, for example, takes one small precaution like washing its dishes at high heat, that would be enough to rule out "Not all restaurants follow food allergy precautions to avoid cross-contamination between multiple dishes.".)

Choice "Limiting cross-contamination from multiple dishes on the same grill is the most effective way to avoid issues for diners with food allergies." is a classic example of an Inference answer choice simply going too far, using "the most effective" when you simply do not have information to rank different precautions.

And choice "Diners with food allergies are generally willing to be patient with longer wait times in order to avoid cross-contamination between dishes." is another example of a choice that might well be true, but does not have any proof in the stimulus.

Example Question #2 : Inference Critical Reasoning

A candy company conducted market research through a survey and a subsequent taste test. In the survey, 27% of respondents said they preferred dark chocolate, 28% said they preferred white chocolate, and 45% said they preferred milk chocolate. But when the same group participated in a taste test of the company's new product line, 60% preferred dark chocolate.

Which of the following can be inferred from the information above?

The survey participants were generally inaccurate regarding their chocolate preferences.

Some people who stated a preference for white chocolate in the survey preferred dark chocolate in the taste test.

Some people who stated a preference for milk chocolate in the survey preferred dark chocolate in the taste test.

Some people who preferred milk chocolate in the taste test had initially stated a preference for white chocolate in the survey.

Most participants expressed a different preference in the taste test than they had indicated in the survey.​

This inference problem forces you to do some math to determine which answer must be true. You know from the given information that some preferences were different between the survey and the taste test (dark chocolate went from 27% to 60%, from the lowest value to the highest, so some people must have changed their preferences from either milk or white chocolate), but each answer choice will require some analysis to determine whether it "could be true" (incorrect answer) or "must be true" (correct).

Choice "The survey participants were generally inaccurate regarding their chocolate preferences." is the qualitative answer and certainly could be true, but isn't necessarily. What if this company simply has lousy white and milk chocolate, but very good dark chocolate? The respondents could have been very accurate in relaying their general preferences, but those preferences just didn't hold in this particular case. So choice "The survey participants were generally inaccurate regarding their chocolate preferences." is incorrect.

Choice "Some people who stated a preference for white chocolate in the survey preferred dark chocolate in the taste test." is more quantitative. It certainly could be true but doesn't have to be. You know that dark chocolate went from 27% to 60%, so it picked up a net gain of 33%. This could be true if some of that gain came from white and some from milk. But since you do not have the taste test totals from white and milk you can play with different combinations. Suppose all who said dark in the survey said dark in the taste test, and then 33% defected from milk to dark. That would leave white unchanged and still give you 60% dark, just with 28% white and now 12% milk. So choice B is not necessarily true and is therefore incorrect.

Choice "Some people who preferred milk chocolate in the taste test had initially stated a preference for white chocolate in the survey." does not have to be true, either. You know that 33% of respondents switched to dark chocolate, but you do not know for certain that anyone switched between white and milk. As you will see with choice "Some people who stated a preference for milk chocolate in the survey preferred dark chocolate in the taste test."...

Choice "Some people who stated a preference for milk chocolate in the survey preferred dark chocolate in the taste test." must be true. You need a net gain of 33% moving from either white or milk to dark. And since only 28% preferred white chocolate, you can't get that 33% gain unless at the very least 5% of people changed from milk to dark.

Choice "Most participants expressed a different preference in the taste test than they had indicated in the survey.​" is incorrect because, again, the minimum change is 33%. All the statements could be true if everyone who liked dark in the survey stuck with dark in the taste test, and then 33% moved to dark from milk. That case satisfies all of the facts but leaves more than half of survey responses intact, thereby invalidating choice "Most participants expressed a different preference in the taste test than they had indicated in the survey.​". Choice "Some people who stated a preference for milk chocolate in the survey preferred dark chocolate in the taste test." is correct.

Example Question #3 : Inference Critical Reasoning

Among the most effective ways to increase sales of an online service is to offer some form of free trial for users to experiment with before they purchase the full service. The benefit of such a practice is to encourage sales in individuals who would not buy the product without having tried it first.

Which of the following is best supported by the information given above?

Because the cost of offering a free trial can be high, companies are often resistant to offering free trials, especially free trials that offer all features included within the paid version of the online service.

The number of sales for a given online service is directly proportional to the number of visitors to the online service's website, a number that tends to increase if a free trial is offered.

In calculating the total number of an online service sold, free trials are generally included as zero-dollar sales rather than as a separate category.

Online services that are easily adapted to free trial versions sell better than do online services that are not readily distributed as free trials.

The number of people who see the free trial as an acceptable replacement for buying the online service is not greater than the number of people who buy the online service because of their experience within the free trial.

As with any inference question, your job here is to understand the information given and to choose an answer choice guaranteed by the text. You are told in this stimulus to this question that free trials are meant to increase sales of the full version of an online service by giving users who would not buy the service without trying it first a chance to experiment with it. Choice "The number of people who see the free trial as an acceptable replacement for buying the online service is not greater than the number of people who buy the online service because of their experience within the free trial." is the only answer choice that is guaranteed by the text. If the number of people who find that the free trial was a good substitute is bigger than the number of people who are incentivized to buy the full online service because of the free trial, then the ability to experiment before you try the full service would not only be meaningless, it would be counter to the reason that companies offer free trials.

Among the other answers, choice "Because the cost of offering a free trial can be high, companies are often resistant to offering free trials, especially free trials that offer all features included within the paid version of the online service." can be eliminated because there is no information about what makes companies more or less likely to offer free trials. Choice "In calculating the total number of an online service sold, free trials are generally included as zero-dollar sales rather than as a separate category." can be eliminated because there is no information given about the spread of companies’ free versus paid sales. Choice "The number of sales for a given online service is directly proportional to the number of visitors to the online service's website, a number that tends to increase if a free trial is offered." can be eliminated because there is no information about whether the two values are directly proportional at all. Choice "Online services that are easily adapted to free trial versions sell better than do online services that are not readily distributed as free trials." can be eliminated for similar reasons to choice "Because the cost of offering a free trial can be high, companies are often resistant to offering free trials, especially free trials that offer all features included within the paid version of the online service.". there is no information about the importance of the ease of creating a free trial.

Example Question #4 : Inference Critical Reasoning

Last year, more copies of accounting software programs were sold than in any previous year. For the first time ever, most of the copies sold were not sold to accountants but rather to individuals doing their own taxes or planning their own family budgets. However, the most-purchased copy of accounting software was a program designed for accountants performing corporate audits.

Which of the following is most strongly supported by the information above?

More non-accountants purchased accounting software last year than in any previous year.

Last year more accounting software was sold to corporations than in any previous year.

Last year there were more copies of accounting software sold to non-accountants than in any previous year.

At least some non-accountants purchased the most-purchased copy of software last year.

Last year fewer copies of accounting software were purchased by accountants than in the previous year.

The answer to this Inference problem is "Last year there were more copies of accounting software sold to non-accountants than in any previous year.". Remember - in an Inference question the correct answer must be true based on the premises, and "Last year there were more copies of accounting software sold to non-accountants than in any previous year." can be proven by the facts. You know that 1) the total number of copies of accounting software was its greatest ever and that 2) the percentage that non-accountants purchased was its greatest ever (the first time over 50%). So non-accountants purchased their greatest-ever share of the greatest-ever total, meaning that they must have purchased their greatest number of copies of accounting software ever.

Among the incorrect answer choices:

"Last year more accounting software was sold to corporations than in any previous year." very well might be true, but cannot be proven. What if the growth in accounting software was entirely due to non-accountants (perhaps this was the first-ever year that a program like TurboTax was available, and so the non-accountant software surged while several accountants went out of business and didn't purchase anything)?

"At least some non-accountants purchased the most-purchased copy of software last year." also could be true, but you certainly cannot prove it. What if the most-sold software was a must-buy for any corporation but had no appeal to individuals?

"More non-accountants purchased accounting software last year than in any previous year." is close, but note the precision in language there: all the premises are about the number of  copies  sold, whereas "More non-accountants purchased accounting software last year than in any previous year." draws a conclusion about the number of  purchasers . What if the number of purchasers stayed the same or even decreased, but each purchaser bought multiple different copies (maybe TurboTax came with a "add on Quicken for a dollar" promotion and almost everyone who purchased one piece of software last year bought two this year?).

And "Last year fewer copies of accounting software were purchased by accountants than in the previous year." of course does not have to be true as there is no proof for it anywhere. You know that the highest total number of copies of accounting software was sold so it is difficult to believe that fewer were sold to non-accountants, and that's the only real evidence you have to get close to this conclusion.

Example Question #5 : Inference Critical Reasoning

Meditation can lead to reduced stress, increased concentration, and a longer life. And contrary to what many skeptics believe, regular meditation is more important than the duration of each session. While longer sessions produce better results, all the benefits listed above are possible from daily meditation sessions that are as short as ten minutes.

Which of the following is best supported by the statement above?

Daily meditation sessions of an hour or longer can increase one's life expectancy.

It is possible to achieve as much of a gain in life expectancy from ten minutes of meditation per day as from less frequent meditation sessions of an hour or longer.

People who meditate for ten minutes each day will live longer than those who meditate less frequently.

Mediation is only effective if it is performed on a daily basis.

Meditating less frequently than once per day will lead to less positive benefits than meditating daily.

With any Inference question, you must select the answer choice that must be true based on the information in the passage. Here, several choices might seem very likely, but the "must be true" standard is crucial for inferences.

Choice "Daily meditation sessions of an hour or longer can increase one's life expectancy." must be true. The premises state that "while longer sessions produce better results, all of the above benefits (including a longer life) are possible from daily-ten minute sessions." From that, you can infer that longer sessions (an hour vs. ten minutes) would at least produce the same benefits, if not better. Additionally, note the easier-to-prove word "can" in "can increase one's life expectancy." This is much easier to prove than "will" or "only," words you see in other answer choices.

Among the other choices, choice "Mediation is only effective if it is performed on a daily basis." goes too far with "only." While the last sentence suggests that daily sessions are effective, the previous sentence uses "regular meditation" (so not necessarily "daily"), and ultimately there is nothing to suggest that even infrequent sessions are completely ineffective.

Choice "People who meditate for ten minutes each day will live longer than those who meditate less frequently." goes too far with the prediction "will" - for one, the argument doesn't give enough information to compare daily ten-minute sessions with, say, five-days-per-week hour-long sessions. But just as damning is the word "will" - predictions are just very hard to prove. Can you conceive of a situation in which people who meditate for ten minutes each day live shorter (too much radiation from their Headspace app?)? If so, "will" is not necessarily true.

Choice "It is possible to achieve as much of a gain in life expectancy from ten minutes of meditation per day as from less frequent meditation sessions of an hour or longer." is wrong for similar reasons as "Mediation is only effective if it is performed on a daily basis." is wrong: the hard fact is that "regular" meditation is more important than the duration of each session, but "regular" does not necessarily mean "daily" so this comparison is impossible to make without further information. For the same reason, choice "Meditating less frequently than once per day will lead to less positive benefits than meditating daily." is also incorrect.

Example Question #6 : Inference Critical Reasoning

Gingivitis is a disease that occurs around the teeth and that can lead to periodontitis, a condition that causes tissue destruction in the gums and even tooth loss. Studies show that diets high in vitamin C can help to both prevent gingivitis and treat periodontitis.

Which of the following is best supported by the statements above?

Increasing the amount of vitamin C in one’s diet has helped some periodontitis patients reduce the severity of that condition.

Periodontitis is a condition only contracted by those who have previously contracted gingivitis.

People whose diets are high in vitamin C are less likely to contract periodontitis than those whose diet are low in vitamin C.

A periodontitis treatment plan that does not include vitamin C is less effective than a plan that does.

Some people suffering from periodontitis do so without having contracted gingivitis.

This Inference problem demonstrates the importance of the "Must Be True" standard for GMAT inferences. With Inference problems, you want to attack the answer choices looking to exploit small flaws, and eliminate accordingly.

Choice "People whose diets are high in vitamin C are less likely to contract periodontitis than those whose diet are low in vitamin C." is too general and emphatic a conclusion. Even though vitamin C itself can help to prevent or treat these conditions, one cannot conclude that those who consume vitamin C will be less likely to contract those conditions. Consider a hypothetical: it could be that vitamin C alone would help, but that vitamin C is often present in sugar-containing foods and most people with high vitamin C levels are also guilty of a high sugar diet that leads to even quicker gum disease. Choice "People whose diets are high in vitamin C are less likely to contract periodontitis than those whose diet are low in vitamin C." may very well be true, but if you can create a hypothetical with a case in which it would not be true, you can eliminate it.

"Periodontitis is a condition only contracted by those who have previously contracted gingivitis." is similar: it seems like it's probably true, since you're told that gingivitis "can lead to periodontitis" but you don't know that it's the only thing that can lead to the condition (as choice "Some people suffering from periodontitis do so without having contracted gingivitis." suggests). Since you're unsure whether gingivitis is the only cause, or just one of multiple potential causes, you can eliminate both "Periodontitis is a condition only contracted by those who have previously contracted gingivitis." and "Some people suffering from periodontitis do so without having contracted gingivitis.".

Choice "A periodontitis treatment plan that does not include vitamin C is less effective than a plan that does." is also not proven. There may be other plans that do not include vitamin C but that are extremely effective. Note the language in the last sentence of the stimulus, that vitamin C "can help to treat periodontitis." "Can help" is soft language that leaves plenty of room for another treatment program to be even more helpful.

Choice "Increasing the amount of vitamin C in one’s diet has helped some periodontitis patients reduce the severity of that condition." is correct, in large part because of similarly soft language. If vitamin C can help treat the condition, that means that it must have helped at least some patients in treatment. That's an easy bar to get over, and since you know for a fact that vitamin C is helpful, you can clear that bar. Choice "Increasing the amount of vitamin C in one’s diet has helped some periodontitis patients reduce the severity of that condition." is correct.

Example Question #7 : Inference Critical Reasoning

Health insurance rates have been steadily increasing in this country for decades. Though health insurance companies paid for a smaller percentage of claims last year than they did ten years ago, the overall rise in the number of claims still means that more money is being paid out, and the companies compensate for this by hiking their rates.

From the information above, it can be inferred that ten years ago

fewer people made health insurance claims than was the case last year.

more claims were not paid by insurance companies than were not paid last year.

health insurance companies paid a greater percentage of their claims than they paid twenty years ago.

the percentage of health insurance claims that were unpaid was less than last year's percentage.

profits made by health insurance companies were similar to profits made by health insurance companies last year.

The stimulus states that "...health insurance companies paid for a smaller percentage of claims last year than they did ten years ago." This means that the companies had a greater percentage of unpaid claims last year. Put another way, they had a smaller percentage of unpaid claims ten years ago, which is what choice "the percentage of health insurance claims that were unpaid was less than last year's percentage." says.

Choice "fewer people made health insurance claims than was the case last year." is incorrect because the number of people making claims is never mentioned. One person can make several claims, so we cannot draw any inferences about the number of people.

"more claims were not paid by insurance companies than were not paid last year." is wrong because the overall number of claims and the percentage of claims not paid were both lower ten years ago. Since they were both lower, their product (Total * Percentage) would also be lower, disproving the statement that more claims were not paid.

Answer "health insurance companies paid a greater percentage of their claims than they paid twenty years ago." is incorrect because we do not have any way of knowing the difference in the percentage of claims paid for these two periods (ten years ago versus twenty years ago).

Answer "profits made by health insurance companies were similar to profits made by health insurance companies last year." is incorrect since we already know that the companies have compensated for paying more real dollars by hiking their rates, but there are too many other factors involved in determining profit to make this comparison.

Example Question #8 : Inference Critical Reasoning

A television news network has recently been criticized for failing to give as much time to individuals who do not believe climate change is occurring as they do to scientists who believe climate change is occurring, even though the network does give equal time to all sides of the debates over immigration, tax reform, and gun policy. The network claims that they only give equal time to both sides of a debate when one side cannot be definitively proven by existing scientific research.

Which of the following can be correctly inferred from the information given above?

The television news network believes that it is important to avoid debate on scientific discoveries.

If the news network gave the same time it gave to scientists to individuals who don’t believe in climate change, it would increase its ratings.

There are no effective counter arguments against climate change that might sway viewers to believe that climate change is not occurring.

The television news network believes that the existence of climate change has been definitely proven by existing scientific research.

No individuals who spoke on the network and who did not believe climate change was occurring were scientists.

Whenever you are asked to make an inference from an argument presented on the GMAT, remember that inferences don't need to be interesting or surprising - they only need to be guaranteed.

The argument here states that a television network has been criticized for not giving as much time to climate change deniers as it does to those who believe in climate change even though they do give equal time to all sides of the debates surrounding other issues like tax reform and immigration. The network claims that this is because they only give equal time if one side of the debate cannot be definitively proven by science.

Since the network does not give equal time to both sides of the climate change debate, that means that people at the network believe that it fits the exception given and that one side (the side that believes in climate change) has been definitively proven by science, which matches answer choice "The television news network believes that the existence of climate change has been definitely proven by existing scientific research.".

Among the other answers, "The television news network believes that it is important to avoid debate on scientific discoveries." can be discarded because while the network does not give equal time for all sides of the debate in some cases, it does not provide a blanket dismissal of debating all scientific discovery. Choice "There are no effective counter arguments against climate change that might sway viewers to believe that climate change is not occurring." can be eliminated since while the network believes the science has been settled, this is not the same thing as claiming that no one will be swayed by the arguments against climate change. Choice "No individuals who spoke on the network and who did not believe climate change was occurring were scientists." is a bit harder to eliminate since the stimulus tells you that the people who believe that climate change is occurring are scientists, but it doesn't specifically say that the individuals who don't believe in climate change aren't scientists, so "No individuals who spoke on the network and who did not believe climate change was occurring were scientists." can be eliminated. Choice "If the news network gave the same time it gave to scientists to individuals who don’t believe in climate change, it would increase its ratings." can also be eliminated since even though some people criticize the network for its current policy, it is unknown how a change in policy would affect overall criticism of the network (and in turn how that would effect ratings).

Example Question #9 : Inference Critical Reasoning

A computer equipped with fingerprint recognition software, which denies access to a computer to anyone whose fingerprint is not on file, identifies a person's fingerprint by analyzing not only the detailed structure of the fingerprint, but also such characteristics as the level of pressure upon which the finger is placed on the scanner and the finger's skin tone. Even the most adept computer hackers cannot duplicate all the characteristics the software analyzes.

Which of the following can be logically concluded from the passage above?

Computers equipped with the recognition software will soon be installed in most financial firms that deal with sensitive electronic information.

It is not possible for any top computer hacker to gain access to a computer equipped with the recognition software solely by virtue of skill in replicating the structure of fingerprints.

The fingerprint recognition software is so sensitive that many authorized users are often denied legitimate access.

Fingerprint recognition software has taken many years and tremendous investment to develop and perfect.

Use of the recognition software is largely impractical due to the time it takes to record and analyze a fingerprint.

The correct answer to this question is "It is not possible for any top computer hacker to gain access to a computer equipped with the recognition software solely by virtue of skill in replicating the structure of fingerprints.". This is an INFERENCE question, requiring the test taker to choose the correct answer that must be true based on the information provided in the stimulus. "Use of the recognition software is largely impractical due to the time it takes to record and analyze a fingerprint." is incorrect as the passage provides no information with regard to the speed of recording and analyzing the fingerprint; as such, no related conclusion can be drawn. "Computers equipped with the recognition software will soon be installed in most financial firms that deal with sensitive electronic information." is incorrect as the passage provides no information with regard to the installation of computers that possess the software in specific locations; as such, no related conclusion can be drawn. "It is not possible for any top computer hacker to gain access to a computer equipped with the recognition software solely by virtue of skill in replicating the structure of fingerprints." This is the correct answer. The passage states that the software detects more characteristics than those that the most successful hackers are able to duplicate; as such, we can conclude it would be impossible for any top hacker to gain access to a protected computer solely by replicating one of multiple characteristics analyzed by the software. "Fingerprint recognition software has taken many years and tremendous investment to develop and perfect." is incorrect as the passage provides no information with regard to the time and investment costs associated with the development of the software; as such, no related conclusion can be drawn. "The fingerprint recognition software is so sensitive that many authorized users are often denied legitimate access." is incorrect as the passage provides no information with regard to errors produced by the software; as such, no related conclusion can be drawn.

Example Question #10 : Inference Critical Reasoning

If the minimum wage increases again, MacDowell’s will have to increase the prices it charges for its products. And if that happens, MacDowell’s has a choice: it can spend more on advertising to attract more customers, or its sales and profitability will decrease. But since the extra advertising costs will simply raise total expenses, increasing those costs will still result in an overall decrease in profitability.

Which one of the following conclusions can be logically drawn from the statements above?

MacDowell’s will see its profitability increase if the minimum wage does not increase.

MacDowell’s will be unable to maintain its current profitability if the minimum wage increases.

Unless the minimum wage increases, MacDowell’s will continue to remain profitable.

If MacDowell’s sees a reduction in its profitability, that means that the minimum wage has increased.

If the minimum wage increases, MacDowell’s will no longer be able to remain profitable.

Because this is an Inference question, the degree of proof for the correct answer is that the correct answer MUST BE TRUE. Because of that:

Choice "Unless the minimum wage increases, MacDowell’s will continue to remain profitable." is incorrect because you're not told what happens if the minimum wage does not increase. This prediction is hard to make, then: suppose the minimum wage stayed flat but a disease was traced to MacDowell's ingredients or a fire burned down its top-grossing store. There are plenty of ways for profitability to be cut even if the minimum wage stays flat.

Choice "If the minimum wage increases, MacDowell’s will no longer be able to remain profitable." is incorrect because it goes too far. You know that profitability will decrease, but not that it will go away entirely.

Choice "MacDowell’s will see its profitability increase if the minimum wage does not increase." is incorrect for similar reasons to choice "Unless the minimum wage increases, MacDowell’s will continue to remain profitable.". There are plenty of factors aside from the minimum wage that could decrease profitability, so choice "MacDowell’s will see its profitability increase if the minimum wage does not increase." is not necessarily true.

Choice "MacDowell’s will be unable to maintain its current profitability if the minimum wage increases." is correct. Because you're told in the argument that, of the two options that would face MacDowell's in the event of a minimum wage increase, both will decrease profitability, you know it to be true that a wage increase will cut profitability.

Choice "If MacDowell’s sees a reduction in its profitability, that means that the minimum wage has increased." is incorrect for similar reasons to "Unless the minimum wage increases, MacDowell’s will continue to remain profitable." and "MacDowell’s will see its profitability increase if the minimum wage does not increase.". Plenty of other factors can lead to a decrease in profitability, so that decrease on its own does not allow you to infer that it was specific to a minimum wage hike.

Tired of practice problems?

Try live online GMAT prep today.

gmat critical thinking sample questions

Report an issue with this question

If you've found an issue with this question, please let us know. With the help of the community we can continue to improve our educational resources.

DMCA Complaint

If you believe that content available by means of the Website (as defined in our Terms of Service) infringes one or more of your copyrights, please notify us by providing a written notice (“Infringement Notice”) containing the information described below to the designated agent listed below. If Varsity Tutors takes action in response to an Infringement Notice, it will make a good faith attempt to contact the party that made such content available by means of the most recent email address, if any, provided by such party to Varsity Tutors.

Your Infringement Notice may be forwarded to the party that made the content available or to third parties such as ChillingEffects.org.

Please be advised that you will be liable for damages (including costs and attorneys’ fees) if you materially misrepresent that a product or activity is infringing your copyrights. Thus, if you are not sure content located on or linked-to by the Website infringes your copyright, you should consider first contacting an attorney.

Please follow these steps to file a notice:

You must include the following:

A physical or electronic signature of the copyright owner or a person authorized to act on their behalf; An identification of the copyright claimed to have been infringed; A description of the nature and exact location of the content that you claim to infringe your copyright, in \ sufficient detail to permit Varsity Tutors to find and positively identify that content; for example we require a link to the specific question (not just the name of the question) that contains the content and a description of which specific portion of the question – an image, a link, the text, etc – your complaint refers to; Your name, address, telephone number and email address; and A statement by you: (a) that you believe in good faith that the use of the content that you claim to infringe your copyright is not authorized by law, or by the copyright owner or such owner’s agent; (b) that all of the information contained in your Infringement Notice is accurate, and (c) under penalty of perjury, that you are either the copyright owner or a person authorized to act on their behalf.

Send your complaint to our designated agent at:

Charles Cohn Varsity Tutors LLC 101 S. Hanley Rd, Suite 300 St. Louis, MO 63105

Or fill out the form below:

Contact Information

Complaint details.

Learning Tools by Varsity Tutors

close

Watch on YouTube!    2 Winner and Loser REAL MBA Resumes - Live Analysis by former AdCom  

Close

Customized for You

Track Your Progress

Practice Pays

Practice thousands of GMAT questions with top expert solutions.

Identify and improve upon mistakes efficiently using our Error Log.

Get the latest tips and news from our top GMAT professionals.

- it’s free and easy!

Thank you for using the timer! We noticed you are actually not timing your practice. Click the START button first next time you use the timer. There are many benefits to timing your practice , including:

We’ll give you an estimate of your score

We’ll provide personalized question recommendations

Your score will improve and your results will be more realistic

gmat critical thinking sample questions

Analysing Real MBA Resumes Selected & Rejected by Top Business Schools

FREE Focus Edition Mock!

Special Offer: Get 20% Off Our GMAT Focus Plans

Memorial Day Savings! Get 44% off on Online Prep and score high

MBA Admissions Consulting Q&A Session: Unlock Top-Ranked MBA Insights!

How to Get a Perfect 805 GMAT Focus Score with Target Test Prep

­The GMAT Club Podcast - My Journey to GMAT 100 Percentile After ISB Rejection

Score 100th Percentile in GMAT Focus Verbal

GMAT RC Masterclass

Conquer Algebra on the GMAT Focus Edition

MBA Spotlight Fair

gmat critical thinking sample questions

08:30 AM PDT

09:30 AM PDT

gmat critical thinking sample questions

08:00 PM PDT

09:00 PM PDT

gmat critical thinking sample questions

12:00 PM EDT

11:59 PM EDT

gmat critical thinking sample questions

05:30 AM PDT

07:30 AM PDT

09:50 AM PDT

11:00 AM PDT

10:00 AM PDT

gmat critical thinking sample questions

01:00 PM EDT

gmat critical thinking sample questions

11:00 AM IST

01:00 PM IST

12:00 PM PDT

Do Not Attempt to "Pre-Think" in Verbal Critical Reasoning Questions

User avatar

The post is bookmarked successfully

IMAGES

  1. How to Master Every Critical Reasoning Question Type

    gmat critical thinking sample questions

  2. GMAT Critical Reasoning

    gmat critical thinking sample questions

  3. GMAT Verbal Reasoning

    gmat critical thinking sample questions

  4. GMAT Critical Reasoning

    gmat critical thinking sample questions

  5. How To Answer Critical Reasoning Questions In GMAT?

    gmat critical thinking sample questions

  6. GMAT Critical Reasoning Questions Practice Test 1

    gmat critical thinking sample questions

VIDEO

  1. GMAT Critical Reasoning| Learn Prethinking |Practice Critical Reasoning Questions

  2. No Critical thinking for Critical Reasoning

  3. GMAT

  4. GMAT Critical Reasoning

  5. GMAT Critical Reasoning Weaken

  6. Mastering CR with Pre-thinking

COMMENTS

  1. Official Sample GMAT Critical Reasoning Questions

    Official Sample GMAT Critical Reasoning Questions. We've already covered why studying with official practice questions is the best way to prepare for the GMAT.But even if you come up with the correct answer to an official problem, you still might not understand the underlying principles used to create that particular question, leaving yourself open to traps and pitfalls set by the test writers.

  2. Free GMAT Practice Questions with detailed Explanations

    Our Free Practice Questions are designed to give you the thorough understanding of how to go about solving a problem that you crave. Our thorough explanations show you what to expect from each GMAT question, detailing question-specific hurdles and common traps. Thankfully, our practice questions provide a wide variety of question types spanning ...

  3. GMAT Critical Reasoning

    GMAT Critical Reasoning Practice Questions- List. Here is a list of all the CR practice questions discussed in this article, for your quick reference: Practice Question 1: Inference. Practice Question 2: Assumption. Practice Question 3: Evaluate. Practice Question 4: Strengthen.

  4. GMAT Critical Reasoning

    GMAT Critical Reasoning questions are designed to evaluate a specific set of attributes relating to logic, and most students preparing for the test have never been exposed to this type of argument analysis. Broadly speaking, there are 3 types of Critical Reasoning questions you will encounter on the GMAT Verbal section:

  5. Tackling GMAT Critical Reasoning Questions

    The easiest mistake to make on GMAT CR is this: you read the question, and an answer pops into your head, perhaps because you've done some "pre-thinking.". You immediately notice that, say, answer choice (B) sounds like whatever you were thinking. So you choose (B), and you don't really read (C), (D), or (E).

  6. GMAT Critical Reasoning Practice Questions

    Click here for a video answer and explanation to the first of our GMAT Critical Reasoning Questions! Click here for a text answer and explanation to GMAT Critical Reasoning Question 1. A minor league baseball franchise experienced a drop in attendance this week after they suffered three losses by margins of ten runs or more last week.

  7. 8 Expert Tips for GMAT Critical Reasoning Questions

    Try to start taking notice of "clue words" in your critical reasoning GMAT practice. For the most part, avoid answer choices that contain language like 'only,' 'never,' 'always,' 'all,' 'none,' 'best,' 'worst,' 'must,' etc. They're nearly always incorrect. For example, in answer choice C in the inference ...

  8. GMAT Critical Reasoning : Practice Tests and Information

    How to approach critical reasoning questions. Read the argument carefully. Wherever possible, identify premises, assumptions and conclusion. (Take approx. 30 seconds to understand.) Take a few extra seconds to make sure you understand the conclusion of the argument. Read the question prompt to find what you are asked to do (strengthen / weaken ...

  9. Free GMAT Critical Reasoning Test

    Our free GMAT Critical Reasoning practice test is a great place to start your test prep. Directions: Each of these GMAT critical reasoning practice questions are based on a short argument, a set of statements, or a plan of action. For each practice question, select the best answer of the choices given. Question 1.

  10. GMAT Critical Reasoning

    About half of the 23 questions that appear in the GMAT verbal section are critical reasoning questions.Typical questions that appear in critical reasoning (GMAT CR) include strengthening an argument, weakening an argument, identifying the assumption of an argument, mimic the reasoning, identify the reasoning, conclusion, inference, bold faced statements, and evaluating an argument.

  11. Critical Reasoning

    Critical Reasoning. Critical Reasoning (CR) questions test your logical reasoning abilities, particularly in terms of making and evaluating arguments. CR questions offer a short paragraph (called a stimulus), typically less than 100 words in length, followed by a question stem and five answer choices.

  12. Critical Reasoning- Collection of All Official Guide Questions

    The table below lists all the Critical Reasoning questions that have appeared in various editions of Official Guides. The table offers the following functionalities. Q#. Source. Link. Type. Difficulty. 1.

  13. Introduction to GMAT Critical Reasoning

    The eight broad categories of GMAT Critical Reasoning questions are: 1) weaken the argument /find the flaw in the argument. 2) strengthen the argument. 3) find the assumption. 4) draw inference/conclusion. 5) structure of the argument, including boldface structure questions and dialogue structure questions. 6) paradox. 7) evaluate the conclusion.

  14. GMAT Prep From Platinum GMAT

    We would like to show you a description here but the site won't allow us.

  15. GMAT Sample Questions

    GMAT practice questions handpicked by our test experts. Get the GMAT math and verbal practice you need to score big. ... GMAT Critical Reasoning Questions. 1. ... For example, it may be necessary to consider what questionable assumptions underlie the thinking and what other explanations or counterexamples might weaken the arguments conclusion ...

  16. Free Sample GMAT Practice Questions

    Free GMAT sample questions for test preparation: quantitative and verbal practice for GMAT exam with detailed answers and explanations to boost your test score! ... Critical Reasoning delves into your ability to analyze and evaluate arguments, requiring strong critical thinking skills. Typically, a short passage lays out an argument, and you ...

  17. Top 10 Tips for GMAT Critical Reasoning

    Here are our 10 best GMAT Critical Reasoning tips. You'll find tips for studying Critical Reasoning before test day, as well as maximizing your chances of getting tough questions right. 1. Don't start at the beginning. A normal GMAT Critical Reasoning argument contains a conclusion and premises that support the conclusion.

  18. Introduction to GMAT Critical Reasoning

    The eight broad categories of GMAT Critical Reasoning questions are: 1) weaken the argument /find the flaw in the argument. 2) strengthen the argument. 3) find the assumption. 4) draw inference/conclusion. 5) structure of the argument, including boldface structure questions and dialogue structure questions. 6) paradox. 7) evaluate the conclusion.

  19. Complete 1000 GMAT Critical Reasoning Series (CR-1000 Series) Question

    Given Kudos: 44. GMAT 1:730 Q51 V37. Send PM. Re: Complete 1000 GMAT Critical Reasoning Series (CR-1000 Series) Question [ #permalink ] Fri Jun 05, 2020 9:55 pm. 1. Kudos. I feel its better to practice from the official guide or use LSAT problems . A lot of ambiguity in the problems above . L.

  20. Free GMAT Practice Questions with detailed Explanations

    You have not answered any question so far. There are 50 free practice questions in our database in total, which you can answer and will improve your skills. You can answer all questions in a row (click on "All Questions") or only all questions of a particular section (click on that Section) or a single selected question (click on that Question).

  21. Sample Questions

    Exam Structure Exam Content Sample Questions Test Security GMAT History. Before you register for the GMAT Focus Edition, get a preview of the types of questions you'll encounter on the exam. Remember: You can learn more about section and question type at Exam Content.

  22. The Most Popular Critical Reasoning Practice Questions

    Join us in GMAT Verbal Practice quiz. Solve the challenging GMAT Critical Reasoning practice questions with peers in timed conditions, analyze your GMAT prep...

  23. How to Tackle Critical Reading Assumption Questions

    Luckily, arguments on GMAT Critical Reasoning questions are relatively formulaic, so let's go over the basics first: A premise is the starting point of the argument. The conclusion is what the author wants you to believe by the end of the argument. The assumption is the missing link between the premise and conclusion.

  24. Inference Critical Reasoning

    Example Question #2 : Inference Critical Reasoning. A candy company conducted market research through a survey and a subsequent taste test. In the survey, 27% of respondents said they preferred dark chocolate, 28% said they preferred white chocolate, and 45% said they preferred milk chocolate. But when the same group participated in a taste ...

  25. Do Not Attempt to "Pre-Think" in Verbal Critical Reasoning Questions

    And yes, to better prepare our students for challenges of the GMAT Focus, we use this data to engineer trap answer choices in our CR practice questions. Remember, CR questions test your ability to analyze the logic of an argument or plan, not your ability to find a justification for your preconceived notion of what the answer should be.